HESI 4 REVIEW - CRITICAL CARE - MCA 3

Lakukan tugas rumah & ujian kamu dengan baik sekarang menggunakan Quizwiz!

A client with a demand pacemaker has a telemetry tracing with a pacing spike but no corresponding QRS complex. The client's myocardium is illiciting a QRS after a delay of several seconds. Which telemetry interpretation should the nurse conclude? Loss of capture. Ventricular fibrillation. Capture from an ectopic focus. A normal finding with a demand pacer.

Loss of capture. Correct If the pacemaker is a demand pacer, a native QRS will be present and the pacer will not fire. If the ventricular automaticity is delayed, the pacemaker will fire. If there is a pacemaker spike that does not producing a QRS, the assessment of the pacemaker is failure to capture.

Which is an example of an internal disaster in a hospital? A hurricane Oil spill from a marine oil tanker Loss of communications capabilities Malfunction of a nuclear reactor with radiation exposure

Loss of communications capabilities Rationale Loss of critical utilities, such as communications capabilities, is an internal disaster. A hurricane is an external natural disaster. An oil spill from a marine oil tanker and malfunction of a nuclear reactor with radiation exposure are examples of technological external disasters

HESI Patient Reviews with RN Case Studies and Practice Test

______________________________________________________________________

A person on the beach sustains a deep partial-thickness sunburn. Which first-aid measure would the nurse recommend before the client seeks health care? Cool, moist towels Dry, sterile dressings Analgesic sunburn spray Vitamin A and D ointment

Rationale Cool, moist towels will decrease edema and minimize pain. Dry dressings, when removed, may further damage the burn site. Although pain is temporarily alleviated, removal of the spray is necessary before further treatment can be instituted; removal may cause injury. Ointments are contraindicated on burns because they have an oil base.

The nurse is caring for a client who is demonstrating signs of impending death. The family is experiencing emotional distress as the client's condition declines. Which information should the nurse provide the family to facilitate the process? Encourage the family to give the client permission to die. Revoke the "do not resuscitate" advanced directive. Send the family to an area to seek spiritual comfort. Give the client pain medication during the end of life hours.

Encourage the family to give the client permission to die. Correct Family members often have difficulty letting go of a dying family member. The nurse should encourage the family to give the client permission to die.

The postanesthesia care unit nurse providing care for a client who had a craniotomy for a meningioma would monitor for which specific clinical manifestation in this client? Dehydration Blurred vision Wound infection Narrowing pulse pressure

Blurred vision Rationale Blurred vision is a sign of increasing intracranial pressure, which may follow a craniotomy. Dehydration, wound infection, and narrowing pulse pressure require time to develop; these clinical manifestations will not be observable immediately after surgery. The pulse pressure widens with increased intracranial pressure.

The nurse is analyzing an arterial blood gas of a client who is mechanical ventilated. The ABG results are pH- 7.42; paCO 2- 50 mmHg; HCO 3- 30mEq/liter. How should the nurse interpret this blood gas? Fully compensated respiratory acidosis. Fully compensated respiratory alkalosis. Fully compensated metabolic acidosis. Fully compensated metabolic alkalosis.

Fully compensated respiratory acidosis. Correct The normal arterial blood gas (ABG) levels are pH: 7.35-7.45; paCO2: 35-45 mmHg; HCO3: 22-26 mEq/liter. In fully compensated respiratory acidosis the pH is within normal limits because compensation has occurred. In compensation, the opposite of the disorder compensates to bring the pH to normal range. In this case, the HCO3 is elevated to compensate for the paCO2

A client placed on hospice care is admitted for palliative radiation treatments to the neck. Which assessment should the nurse identify as a priority? Pain assessment. Respiratory assessment. Cardiovascular assessment. Integumentary assessment.

Pain assessment. Correct Frequent pain assessments are the most important interventions in delivering end-of-life care for a client. The goal of end-of-life treatment is to manage a client's symptoms to provide a pain-free and stress-free environment to improve a client's quality of life.

Two days following cardiac bypass surgery, the nurse places a client's mediastinal chest tube to water seal. The client is using the incentive spirometer hourly while awake. Which assessment finding warrants intervention by the nurse? Serosanguineous fluid in collection container. Fluid fluctuation in tubing with respirations. Water seal level2 cm below the water seal fill line. Reportof chest tube insertion site tenderness.

Water seal level2 cm below the water seal fill line. Correct If the water seal level is above or below the prescribed water seal line, it will increase the client's work of breathing which will increase the risk of a pneumothorax.

The cardiac monitor alarms and the nurse finds a client with no palpable carotid pulse and no spontaneous respirations. The cardiac monitor displays a normal sinus rhythm. Which intervention should the nurse implement? Assess for signs of cardiac tamponade. Begin chest compressions at 120 per minute. Check for responsivenesswith sternal rub. Obtain a STAT 12 lead electrocardiogram.

Begin chest compressions at 120 per minute. Correct The client is in a pulseless electrical activity (PEA) rhythm. No pulse or respirations require immediate chest compressions. The client may have developed a cardiac tamponade because in absence of hypovolemia and a tension pneumothorax, PEA is suggestive of a possible cardiac tamponade. However, the nurse's first action should be chest compressions.

Which action of the emergency department nurse caring for a group of clients injured in a community disaster would need correction? Triaging the victims Supervising volunteers Providing on-site first aid Removing people from danger

Removing people from danger Rationale During a community disaster, removing people from danger is done by firefighters and other disaster trained emergency personnel. Nurses would not be involved in this process. After the removal of people from danger, the nurses triage the victims under triage categories. The nurses supervise volunteers and provide on-site first aid to victims.

A client with chronic kidney disease (CKD) is admitted for strict fluid restriction. Which assessment finding requires additional nursing action? Crackles in the lungs. Decreased serum creatinine level. Increased weakness. Increase in serum potassium.

Crackles in the lungs. Correct In CKD, strict fluid restriction is vital to minimize the progression of fluid overload. Fluid restriction that is insufficient to reduce circulatory volume overload can manifest as pulmonary crackles or coarse rales in the lungs that compromises oxygenation and requires additional nursing action.

Which color tag is assigned the lowest priority for care in a mass casualty event? Red Black Green Yellow

Black Rationale Clients with black tags are expected to die or are already dead; therefore, these clients are the lowest priority. The first priority should be given for clients with a red tag because the client's life may be saved with immediate treatment. A client with a yellow tag should be given second priority because she or he can wait for the treatment for some time. A client with a green tag can be given care after some time, because her or his condition would be stable.

Which problem is suggested when a client at 37 weeks' gestation experiences a sudden sharp pain in her abdomen with a period of fetal hyperactivity followed by fundal tenderness and a small amount of dark-red bleeding? True labor Placenta previa Partial abruptio placentae Abdominal muscular injury

Partial abruptio placentae Rationale Typical manifestations of abruptio placentae are sudden sharp localized pain and small amounts of dark-red bleeding caused by some degree of placental separation. True labor begins with regular contractions, not sharp localized pain. There is no pain with placenta previa, just the presence of bright-red bleeding. There are no data to indicate that the client sustained an injury.

An older client is admitted to the intensive care unit after a small bowel resection. The postoperative prescriptions include a patient-controlled analgesia (PCA) device with morphine titrated per protocol. Which information should the nurse provide the client about the use of the PCA? Push button when pain is first experienced instead of waiting until pain is unbearable. Family members or visitors can press the button when the client grimaces in pain. Press the button every 15 minutes even when pain is not present.

Push button when pain is first experienced instead of waiting until pain is unbearable. Correct A client should be awake to self administer PCA analgesia, which is titrated and programmed to prevent overdosing. The client should press the PCA button on demand when pain is first experienced to prevent escalation of pain that can become uncontrollable

A client is admitted to the intensive care unit with urosepsis. Which findings should the nurse report to the healthcare who suspects the client is at risk systemic inflammatory response syndrome (SIRS)? Temperature 102 F (38.9 C), PaCO 2 28, and apical pulse 100 beats per minute. Temperature 101 F (38.3 C), PaCO 2 55, apical pulse 80 beats per minute. Temperature 98.7 F (37.1 C), white blood cell count 5.5 cells/mm 3, respiratory rate 20 breaths per minute. Temperature 100.2 F (37.9 C), white blood cell count 10.0 cells/mm 3, respiratory rate 18 breaths per minute.

Temperature 102 F (38.9 C), PaCO 2 28, and apical pulse 100 beats per minute. Correct Diagnositic Criteria for systemic inflammatory response syndrome (SIRS) includes two out of four criteria are present in abnormal vital signs, arterial blood gases, and white blood cell counts. The nurse should notify the healthcare provider about the client's findings of temperature 102oF (38.9oC), PaCO2 28, and apical pulse 100 beats/minute.

When the triage nurse assigns a green tag to a client during triage after a mass casualty event, which statement is true about the client's injuries? The client is expected and allowed to die. The client has an immediate threat to life. The client has major injuries that require treatment. The client has minor injuries that do not require immediate treatment.

The client has minor injuries that do not require immediate treatment. Rationale The green tag is assigned to the client who has minor injuries that do not require immediate treatment. The client who is expected and allowed to die will be given a black tag. The client who has an immediate threat to life will be given a red tag. The client who has major injuries that require treatment is given a yellow tag.

Based on their conditions, which client would be triaged first according to the 3-tiered triage system? A - CHEST PAIN B - MULTIPLE FRACT C - RENAL COLIC D - STRAINS SPRAINS

A - CHEST PAIN Rationale Client A reporting chest pain is considered to have a life-threatening condition and is triaged as emergent. Clients B and C having displaced or multiple fractures and renal colic need quick treatment but are not considered to have immediately life-threatening conditions, particularly when compared with client A's condition. They are triaged in the urgent tier level. Client D with strains and sprains is triaged as non-urgent because this client has strains and sprains and can wait for treatment.

Which of these clients in the emergency department would the nurse see first? A client with a head injury A client with a fractured femur A client with ventricular fibrillation A client with a penetrating abdominal wound

A client with ventricular fibrillation Rationale Ventricular fibrillation will cause irreversible brain damage and then death within minutes because the heart is not pumping blood to the brain. Defibrillation, or cardiopulmonary resuscitation until defibrillation is possible, must be initiated immediately. Although head injury, fractured femur, and penetrating abdominal wound require prompt treatment, death is not as imminent as with ventricular fibrillation.

The client with which tag color requires immediate treatment in the emergency department according to disaster triage tag system? A - YELLOW B - RED C - GREEN D - BLACK

B - RED Rationale According to the disaster triage tag system, a red tag is applied to the clients who require immediate treatment. The client B requires immediate treatment. A yellow tag is applied to clients who can wait a short time for treatment (30 minutes to 2 hours). Green tags are applied to clients who can ambulate on their own and do not have serious injuries. Black tags are applied to the clients who are dead or expected to die.

Which color tag will be given by the triage nurse to a client assigned to class IV during a mass casualty situation? Red Black Green Yellow

Black Rationale A client considered class IV during triage in a mass casualty situation will be given a black tag. The red tag is associated with class I, green tag with class III, and yellow tag with class II.

A client reports to the nurse feeling achy and weak, being tired and coughing all the time, frequent headaches and experiencing night sweats. The client's assessment is significant for crackles scattered throughout the lungs, dependent peripheral edema +3/+4, S3 and S4 heart sounds, temperature of 102.4° F(39.1° C), heart rate of 110 beats/minute, respirations of 20 breaths/minute, and blood pressure of 105/60 mmHg with a mean arterial pressure of (75). Which diagnostic procedure should the nurse prepare to do first? Metabolic panel with electrolytes. Complete blood count. Liver function test. Blood culture.

Blood culture. The client is demonstrating clinical signs and symptoms of infective endocarditis. The key in treating infective endocarditis is identifying the causative infectious agent and treat with the appropriate antibiotics. Blood cultures should identify which bacteria is the offending bacteria causing the endocarditis. What distinguishes infective endocarditis from the other conditions listed is the presence of the heart failure symptoms of edema, and S3 and S4 heart sounds.

Which type of damage is most likely caused by a crush injury to victims of a terrorist attack involving explosive devices? Middle ear injury Blunt trauma to the head Shrapnel injury in the abdomen Damaged lungs due to shock wave

Blunt trauma to the head Rationale Crush injuries often result from explosions in confined spaces causing structural collapse, such as falling debris. Blunt trauma to the head is an example of a crush injury. Middle ear injury is a common type of blast injury that results from the supersonic over pressurization shock wave caused by the explosion. Some explosive devices contain materials that are projected during the explosion, leading to penetrating injuries. Shrapnel injury in the abdomen is a type of penetrating injury. Lung damage due to a shock wave is a type of blast injury.

What is the nurse's priority action for a client with this ECG tracing? Call the rapid response team and start compressions. Notify the provider for further orders. Have the client perform Valsalva maneuvers. Document the rhythm as the only finding.

Call the rapid response team and start compressions. Correct The ECG tracing rhythm is indicating ventricular fibrillation where the heart muscle cells are fibrillating and the ventricle are not contracting effectively to provide cardiac output and eject blood into the circulatory system. After checking the client's pulse to verify no pulse, activate the rapid response team and begin chest compressions.

A client who returns to the postoperative unit after a total thyroidectomy suddenly becomes short of breath and develops stridor. What action should the nurse implement first? Call the rapid response team for emergency assistance. Encourage the client to relax as respiratory effort eases. Document the findings and monitor the client hourly. Call respiratory therapy to provide cool mist oxygen per mask.

Call the rapid response team for emergency assistance. Correct A client in the immediate postoperative period after a thyroidectomy can have bleeding into the soft tissue around the incision site that obstructs the airway and causes an inspiratory stridor and laryngospasm. The client is at risk for a life-threatening event, and the nurse should first call the rapid response team for emergency management.

The client admitted to the emergency department after an earthquake in the city with which level of resource intensity would the nurse triage into emergency severity index level 1 (ESI-1)? A - High resource intensity B - HIGH RESOURCE INTENSITY, STAFF AT BEDSIDE CONT C - receiving medium to high resource intensity D - low resource intensity involving one simple diagnostic

Client B Rationale High resource intensity is required for clients with severe life-threatening conditions such as cardiac arrest and severe respiratory depression. Client B should be triaged as ESI-1. Client A's condition involves high resource intensity and frequent monitoring but without staff at the bedside continuously and was triaged as ESI-2. Client C, receiving medium to high resource intensity and who has complex multiple procedures and diagnostic studies, should be triaged at ESI-3. Client D, using low resource intensity involving one simple diagnostic study and procedures, should be triaged as ESI-4.

A client is admitted to the intensive care unit with pulmonary edema. Which clinical finding would the nurse expect when performing the admission assessment? Weak, rapid pulse Decreased blood pressure Radiating anterior chest pain Crackles at bases of the lungs.

Crackles at bases of the lungs Rationale Crackles are the sound of air passing through fluid in the alveolar spaces; in pulmonary edema, fluid moves from the intravascular compartment into the alveoli. Hypervolemia leads to pulmonary edema. The pulse is bounding with hypervolemia. The blood pressure usually is increased with hypervolemia. Radiating anterior chest pain occurs with angina or a myocardial infarction.

The nurse is planning to triage clients after a disaster. Which client does the nurse categorize into the green-tagged category? A - AIRWAY OBTS B - OPEN FRACT C - MASSIVE HEAD TRAUMA D - BRUISES LACERATIONS ON SKIN

D - BRUISES LACERATIONS ON SKIN Rationale The disaster triage tag system categorizes triage priority by color. Clients with minor injuries that can be managed in a delayed fashion are categorized as green-tagged. Client D with bruises and lacerations on the skin is green-tagged. Client A, with the life- threatening condition of an airway obstruction, is red-tagged. Client B with large wounds and open fractures needs treatment within 30 minutes to 2 hours and is yellow-tagged. Client C with critical massive head trauma is black-tagged.

The nurse is caring for a client who is 4 days postoperative for abdominal surgery. The client reports acute onset of difficulty breathing. The nurse obtains the vital signs which include a heart rate of 120 beats/minute and respirations of 35 breaths/minute. Which diagnostic test should the nurse prepare the client to expect the healthcare provider to prescribe? D-dimer blood test. Coagulation time and platelet count. Echocardiogram. Mass spectrometry.

D-dimer blood test. Correct Pulmonary embolism (PE) is a postoperative complication due to a peripheral thrombus breaking off and becoming embolic and lodging in the lung microcirculation causing acute difficulty breathing, tachypnea, and tachycardia. The nurse should prepare the client for a D-dimer test which measures the levels of a substance in the bloodstream when a blood clot breaks down

The nurse is caring for a client who is receiving mechanical ventilation for acute respiratory distress syndrome (ARDS). The ventilator is alarming continuously indicating high peak pressures for the client. Which pathologic changes in the client is causing the ventilator alarms? Decreased lung compliance. Increased respiratory rate. Low volume of expired air. High tidal volumes.

Decreased lung compliance. Correct The inflammatory response in the ARDS causes changes in lung compliance. ARDS causes a decreased lung compliance, or stiffer lung, which causes the high peak airway pressures and frequent ventilator alarms.

Where will the nurse place the V 1 lead when obtaining a 12-lead electrocardiogram? Fifth intercostal space, left midaxillary line Second intercostal space, left sternal border Fourth intercostal space, right sternal border Fifth intercostal space, left midclavicular line

Fourth intercostal space, right sternal border Rationale Positions for these 6 leads are as follows: V 1: fourth intercostal space, right sternal border; V 2: fourth intercostal space, left sternal border; V 3: halfway between V 2 and V 4; V 4: fifth intercostal space, left midclavicular line; V 5: fifth intercostal space, left anterior axillary line; V 6: fifth intercostal space, left midaxillary line.

The nurse is caring for a client who is prescribed a potassium-sparing diuretic and has a potassium level of 6.1 mEq/L (6.1 mmol/L). Which intervention should the nurse perform? Obtain a 12-lead electrocardiogram (ECG). Call a rapid response. Insert an intravenous (IV) line. Schedule a cardiac catheterization.

Obtain a 12-lead electrocardiogram (ECG). Correct An elevated potassium level (normal 3.5 to 5.0 mEq/L (3.5 to 5.0 mmol/L) can lead to rhythm changes of the heart's conduction system. The nurse should obtain a 12-lead ECG to evaluate the impact of hyperkalemia on the cardiac waveform and rhythm.

Which conditions are given a green tag according to triage disaster tag system? Select all that apply. One, some, or all responses may be correct. Strains Abrasions Open fractures Airway obstruction High cervical spinal cord injury

Strains Abrasions Rationale A green tag is issued for clients who are nonurgent or classified under class III. Strains and abrasions are considered minor injuries and nonurgent. Walking wounded clients with minor injuries are triaged under this category. Clients with open fractures and who can wait some time for care are issued yellow tags and triaged under class II. Clients with airway obstruction are issued red tags and triaged under class I. Clients with high cervical spinal cord injury are issued black tags because they are expected to die and are categorized under class IV.

Which intervention should the nurse perform prior to the removal of an endotracheal tube from a client? Suction the endotracheal tube thoroughly. Pre-medicate the client with pain medication. Increase the FiO 2 for a minimum of 15 minutes. Provide positive pressure ventilation prior to extubation.

Suction the endotracheal tube thoroughly. Correct Prior to extubation of a client and removing the endotracheal tube (ETT), the nurse should ensure good airway clearance by suctioning out the ETT thoroughly.

Which Glasgow Coma Scale score indicates that the client is in a coma? 6 9 12 15

6 Rationale A score of 8 or below indicates coma. The Glasgow Coma Scale is used to assess the extent of neurologic damage; it consists of three assessments: eye opening, response to auditory stimuli, and motor response. Consciousness exists on a continuum from full consciousness to coma. A score can be from 3 to 15; the lower the score, the more indicative of coma. To achieve the ratings of 9, 12, or 15 the client must be exhibiting some meaningful responses.

A client who experienced an intracranial hemorrhage is receiving cistracurium to induce a pharmacological state of paralysis. Which intervention should the nurse implement during the therapeutic paralysis? Administer a sedating medication. Monitor the client for seizures. Titrate the drug dose to prevent addiction. Check the client's gag reflex every 2 hours..

Administer a sedating medication. Correct Cisatracurium is a paralytic agent that does not provide sedation or pain relief. The nurse should give a sedative to provide comfort to the client who is unable to move or physically response to pain or anxiety related to the induced paralysis.

Which triage class client would the nurse plan to treat first according to the disaster triage tag system? Class I Class II Class III Class IV

Class I Rationale Class I clients are emergent clients who are marked with a red tag. These clients have an immediate threat to life and need attention first. Class II clients have major injuries and need treatment within 30 minutes to 2 hours. Class III clients have minor injuries and can be treated in a delayed manner. Class IV clients are those who are expected to die or are dead.

Which component of sea water is known to be responsible for making sea water hypertonic compared with body fluids? Salt Mud Algae Chlorine

Salt Rationale Salt makes sea water hypertonic compared with body fluids. Mud, algae, and chlorine are present in abundance in fresh water, which is hypotonic.

A burn victim has waxy white areas interspersed with pink and red areas on the anterior trunk and all of both arms. Which percentage of total body surface area (TBSA) would the nurse calculate? 20 25 30 36

36 Rationale Using the rule of nines, the percentage of total body surface area burned is 9% for each arm (18% total for both arms) and 18% for the anterior trunk; thus the total body surface area burned is 36%. The choices 20%, 25%, and 30% are too low.

Which intervention is the highest priority nursing action for a client with a scorpion bite? Assessing the client's vital signs Applying an ice pack to the sting site Contacting the poison control center Covering the client with cooling blanket

Assessing the client's vital signs Rationale The first priority is vital sign assessment for a client who sustained a scorpion sting. Continuous monitoring for several hours in the critical care unit helps ensure the client's safety. The nurse has to then apply an ice pack to the sting site to reduce pain. The poison control center assists with client management, particularly in regard to the use of medications for scorpion stings, and this is a medium priority. Covering the client with cooling blanket to reduce fever is of lowest priority.

Which client would be treated first? Client A - AIRWAY OBSTRUCTION Client B - extensive full- thickness burns Client C - open fractures Client D - closed fracture and abrasions

Client A - AIRWAY OBSTRUCTION Rationale Client A with an airway obstruction has an immediate threat to life and the condition is considered emergent, which meets the criteria for class 1. Client B with extensive full- thickness burns should be triaged under class 4 because the client with this condition is expected to die. Client C with open fractures implies a major injury that requires treatment and is considered urgent. This client is triaged under class 2. Client D with a closed fracture and abrasions indicates a minor injury that does not require immediate treatment and is triaged under class 3.

A client diagnosed with gastric ulcers is admitted for a cauterization procedure. Which finding(s) should the nurse report to the healthcare provider? (Select all that apply.) Decreased level of consciousness. Hypoactive bowel sounds. Decreased urine output. Coffee-ground vomitus. Positive guaiac stools.

Decreased level of consciousness. Correct Decreased urine output. Correct Upper GI bleeding can lead to severe blood loss. Hypotension is a late sign of hypovolemic shock. A systolic blood pressure less than 100 mmHg or a heart rate greater than 120 beats/minute is reflective of a blood loss greater than 1000mL. A decline in the level of consciousness or urine output are considered late signs of shock from increased bleeding and the nurse should notify the healthcare provider of a possibility of increased gastric bleeding. The nurse should be assessing the client's vital signs every 15 minutes.

A pregnant client with severe abdominal pain and heavy bleeding is being prepared for a cesarean birth. Which is the priority intervention? Teaching coughing and deep-breathing techniques Cleansing the surgical site and administering an enema Providing a sterile gown and inserting an indwelling catheter Ensuring that an informed consent is obtained and that the client is assessed for medication allergies

Ensuring that an informed consent is obtained and that the client is assessed for medication allergies Rationale In an emergency surgical situation when invasive techniques are necessary, it is essential to have a signed consent on file as well as a history of the client's known allergies. Teaching coughing and deep-breathing techniques is not a priority in an emergency such as this. In an emergency, cleansing of the surgical site is performed in the operating room; an enema usually is not given before a cesarean, especially to a client who is bleeding, because it may stimulate contractions and worsen the hemorrhage.

Which nursing intervention has the highest priority for a client who was in a motor bike accident and has a severe neck injury? Assessing for crepitus Assessing for bleeding Maintaining a patent airway Performing neurological assessment

Maintaining a patent airway Rationale The nurse should assess, ensure, and maintain a patent airway first in a client with neck trauma. The nurse then may palpate the skin near the esophagus to assess crepitus, which indicates an injury to the esophagus. After ensuring airway patency, the nurse should assess for bleeding or impending shock. The nurse should also perform a neurological assessment for mental status, sensory level, and motor function, which holds a medium priority.

The nurse is caring for a client who underwent surgical repair of the aorta after sustaining injuries in a fall. Which finding indicates improved blood flow after the surgery? Movement of lower extremities. Decreased urinary output. Maintained weight. Blood pressure 90/50.

Movement of lower extremities. Correct The nurse should assess for postoperative transient paralysis due to lack of blood flow during the surgical procedure as a result of aortic cross-clamp time. The ability of the client to move the lower extremities indicates that adequate blood flow has been restored.

A client has a diagnosis of partial-thickness burns. While planning care, the nurse recalls that the client's burn is different than full-thickness burns. Which information did the nurse recall? Partial-thickness burns require grafting before they can heal. Partial-thickness burns are often painful, reddened, and have blisters. Partial-thickness burns cause destruction of both the epidermis and dermis. Partial-thickness burns often take months of extensive treatment before healing.

Partial-thickness burns are often painful, reddened, and have blisters. Rationale Pain is from the loss of the protective covering of the nerve endings; blisters and redness occur because of the injury to the dermis and epidermis. Because some epithelial cells remain, grafting is not needed with a partial-thickness burn unless it becomes infected and further tissue damage occurs. Partial-thickness burns involve only the epidermis and only part of the dermis. Recovery from partial-thickness burns with no infection occurs in 2 to 6 weeks.

A client was admitted with full-thickness burns 2 weeks ago. Since admission, the client has lost an average of 1 lb (0.5 kg) of weight each day. Which dietary adjustment would the nurse recommend? Increase low-sodium milk intake. Provide high-protein drinks. Increase foods that are low in potassium. Provide 10% more calories in the form of fats.

Provide high-protein drinks. Rationale High-protein drinks have twice the calories per volume of other fluids and provide protein for wound healing. Low-sodium milk does not contain adequate calories to help meet the high metabolic rate associated with burns. Potassium is restricted during the first 48 to 72 hours after a burn injury, not 2 weeks after the injury. Increased calories in the form of protein and carbohydrates, not fats, are needed.

Which parameters would the nurse consider for proper rapid baseline assessment using a disability mnemonic (AVPU) in a client with drug abuse? Select all that apply. One, some, or all responses may be correct. Level of anxiety Reaction to pain Response to voice Body temperature Evidence of assault

Reaction to pain Response to voice Rationale The disability examination provides a rapid baseline assessment of neurological status. It helps evaluate level of consciousness by the AVPU mnemonic, which also helps assess the responsiveness to pain and voice. Level of anxiety is not assessed by a disability mnemonic. Body temperature and evidence of assault are assessed in a primary survey of exposure.

A client returns to the postoperative unit after arteriovenous graft placement. The telemetry is showing tall, peaked T waves on the waveform. Which action should the nurse implement? Review the client's recent serum potassium level. Prepare the client for synchronize cardioversion. Notify Rapid Response Team for ST-elevation myocardial infarction. Move the telemetry leads to the correct placement on the chest.

Review the client's recent serum potassium level. Correct Hyperkalemia causes the tall, pointed, peaked T waves in the telemetry waveform. The nurse should evaluate the client's recent serum potassium level.

The nurse is collecting a sample for arterial blood gases (ABGs) for a client with hypoxia due to cardiomyopathy. Which should the nurse assess prior to obtaining the arterial blood sample? Ulnar blood flow. Apical heart rate. Oxygenation level. Breath sounds.

Ulnar blood flow. Correct Prior to obtaining a sample for arterial blood gases (ABGs), the nurse should assess ulnar blood flow by performing the Allen test, which evaluates circulation to the wrist. If ulnar blood flow is compromised, the nurse should not obtain ABGs from the artery in the affected wrist

A client who is critically ill requests to receive the Sacrament of the Anointing of the Sick. Which clergy member should the nurse contact? Rabbi. Priest. Sharma. Ayatollah.

Priest. Correct In the Roman Catholic religion, the "Sacrament of the Anointing of the Sick" is a ritual that is performed on followers when they are seriously ill or dying. In the Roman Catholic community and this is one of the seven sacraments followers received throughout their life. The sacrament is a source of grace and strength for the church member and a sign of God's presence. This sacrament can only be administered by an ordained Roman Catholic priest.

A client's membranes rupture during the transition phase of labor, and the amniotic fluid appears pale green. Which intervention, upon delivery of the infant, is a priority? Stimulating crying Administering oxygen Placing the newborn on the mother's chest Providing for suctioning of the oropharynx as the head emerges

Providing for suctioning of the oropharynx as the head emerges Rationale The color of the amniotic fluid is indicative of meconium staining; the practitioner must prepare for the potential for fetal aspiration of meconium. The newborn should not be stimulated to cry until the airway has been cleared of meconium. Oxygen is administered only after a patent airway is established and if needed. Placing the newborn on the mother's chest should be not done until after the airway has been cleared.

Which emergency medical service agency offers service for a group of clients admitted to the emergency unit with mass fatalities? Medical Reserve Corps (MRC) Hospital Incident Command System (HICS) International Medical-Surgical Response Teams (IMSRT) Disaster Mortuary Operational Response Teams (DMORT) .

Rationale The National Disaster Medical System (NDMS) is responsible for managing the Federal Government's medical response to medical emergencies and disasters. One of their additional services is the DMORT. DMORTs manage mass fatalities. The MRC offers its services to health care facilities or the community in a supportive to supplemental capacity during times of need, such as a disaster or pandemic disease outbreak. HICS is a part of National Incident Management System (NIMS) that standardizes disaster operations. IMSRT establishes fully functional field surgical facilities.

The nurse is analyzing an arterial blood gas (ABG) of a client who is mechanically ventilated. The ABG results are pH - 7.17; paCO2 - 70 mmHg; HCO3 - 30 mEq/liter. How should the nurse interpret this blood gas? Respiratory acidosis. Respiratory alkalosis. Metabolic acidosis. Metabolic alkalosis.

Respiratory acidosis. Correct The normal arterial blood gas (ABG) levels are pH: 7.35-7.45; paCO2: 35-45 mmHg; HCO3: 22-26 mEq/liter. In respiratory acidosis, the pH is decreased and the CO2 is increased.

Which area of assessment is included in the Glasgow Coma Scale? Breathing patterns Deep tendon reflexes Eye accommodation to light Response to verbal commands

Response to verbal commands Rationale The three areas of assessment to determine the level of consciousness using the Glasgow Coma Scale are motor response to verbal commands, eye opening in response to speech, and verbal response to speech. Assessing breathing patterns, deep tendon reflexes, and eye accommodation are not included in the Glasgow Coma Scale.

The nurse is caring for a client admitted to the intensive care unit with a traumatic brain injury from a motor vehicle collision. The client is experiencing increased intracranial pressure (ICP). The healthcare provider explains to the family that the client needs to go to surgery for decompressive craniectomy. Which information should the nurse explain to the client? An over-lying cranial bone flap is removed to allow swelling brain tissue to expand. The procedure uses a magnetic resonce imaging-guided laser ablation. An opening into the skull is made to remove damage tissue. A burr hole is drilled through the cranil bones to evacuate blood.

An over-lying cranial bone flap is removed to allow swelling brain tissue to expand. Correct The nurse should explain to the client that a decompressive craniectomy removes an overlying bone flap to allow for underlying brain tissue to expand and swell without being compressed by the cranial vault.

Which client condition would require the highest priority for treatment among four clients admitted at the same time under mass casualty conditions? A - MASSIVE HEAD TRAUMA B - OPEN FRACTURE WITH DISTAL PULSE C - SHOCK D- STRAINS AND CONTUSIONS

Client C RATIONALE: Client C reporting with shock is identified with a red tag, indicating an immediate threat to life. The client with massive head trauma is labeled with a black tag, which indicates the client is deceased or is expected to die, so treatment will not be provided to maximize resources to save the most clients possible. The client reporting with an open fracture with distal pulse is triaged as class 3, in which the treatment is considered urgent but less so than for a red-tagged client. Strains and contusions are minor injuries and are considered nonurgent; client D should be triaged as class 3.

Applying the emergency severity index (ESI) criteria, which client condition is considered least severe? Cardiac arrest Abdominal pain Multiple trauma Closed extremity trauma

Closed extremity trauma Rationale Care for a client with closed extremity trauma could be delayed because it is considered less severe when compared with other client conditions and triaged in emergency severity index (ESI-4). This client is given least priority. The client with cardiac arrest is triaged under ESI-1 and should be seen immediately because the condition is more severe. The client with abdominal pain is triaged under ESI-3 and should be seen within 1 hour. The client with multiple trauma should be seen within 1 hour and is triaged under ESI-2.

Which nursing intervention is the most effective in preventing a seizure in a client with severe preeclampsia? Providing a plastic airway Controlling external stimuli Having emergency equipment available Keeping calcium gluconate at the bedside

Controlling external stimuli Rationale Reducing lights, noise, and stimulation minimizes central nervous system irritability, which can trigger a seizure. A plastic airway will not prevent a seizure. Available emergency equipment will not prevent a seizure, although oxygen and suction equipment may be useful after a seizure. Calcium gluconate is the antidote for magnesium sulfate toxicity; it does not prevent seizures.

Which client requires lowest priority according to the Glasgow Coma Scale? EYE - MOTOR - VERBAL A - 1, 2, 3 B - 2, 3, 3 C - 3, 5, 3 D - 4, 5, 5

D - 4, 5, 5 4+5+5= 14 GCS Rationale Spontaneous eye opening receives 4 points; 5 points for motor response indicates the client has localized pain and has oriented verbal response. A score of 14 indicates that client D requires the lowest priority and that treatment can be delayed for some time. Client A, who has absence of eye reflexes (1), extended motor reflex (2), and inappropriate speech (3), requires high priority. Client B, who has pain opening the eyes (2) and abnormal motor response (3) and speech (3), requires emergent treatment. Client C, who has moderate eye movement (3), has localized pain (5), and speaks inappropriate words (3), requires urgent treatment.

Which type of emergency assessment is the nurse performing when the nurse reassesses level of consciousness and mental status in a client who survived a fire? Airway Disability Breathing Full set of vital signs

Disability Rationale Reassessing the level of consciousness and mental status in a client would provide baseline assessment of neurological status, which is an aspect of emergency assessment of disability performed in a primary survey as a life-saving intervention. Emergency assessment of the airway is performed to assess respiratory distress, bleeding, and edema. Emergency assessment of breathing is performed to assess normal airway passage, tension pneumothorax, or the need for mechanical ventilation. Assessing the full set of vital signs involves monitoring temperature, heart rate, and respiratory rate.

What action should the nurse implement for a client with this ECG tracing. Document the rhythm. Notify the provider immediately. Administer drugs per Advanced Cardiac Life Support protocol. Call the family and ask if a chaplain should be summond.

Document the rhythm. Correct The ECG tracing rhythm shown is normal sinus rhythm. The nurse should document the interpretation of the rhythm in the client's electronic health record as per medical institution's protocol. The ECG tracing rhythm strip represents a three second strip, there are (4) normal complete "QRS" complexes present on the ECG rhythm strip, indicating normal sinus rhythm (NSR) with a heart rate of 80 beats/minute.

Which color-tagged clients usually make up the greatest number in most large-scale multi-casualty situations, based on the disaster triage tag system? Red Black Green Yellow

Green Rationale Green-tags clients usually make up the greatest number in most large-scale multicasualty situations. These clients have minor injuries and they may actually evacuate themselves from the mass casualty scene and go to the hospital in a private vehicle. Red-tagged clients have major injuries, black-tagged clients are expected and allowed to die, and yellow- tagged clients have major injuries. Clients belonging to these three categories usually do not make up the greatest number in most large-scale multicasualty situations.

Which type of emergency assessment is being performed when the nurse is inserting a gastric tube and arranging for diagnostic studies for a client who sustained injuries after a bus accident? Disability Breathing Focused adjuncts Giving comfort measures

Focused adjuncts Rationale Emergency assessment of focused adjuncts is performed in a secondary survey. Inserting a gastric tube and arranging for diagnostic studies are included in this type of assessment. Emergency assessment of disability is performed in a primary survey to identify a client's level of consciousness. Emergency assessment of breathing is performed in a primary survey to identify respiratory distress or any need for mechanical ventilation. Giving comfort measures is performed during the secondary survey to provide emotional support to the client and caregiver.

According to the paramedic's report, the victim of a motor vehicle collision was sitting in the passenger seat on the left side of the vehicle. The vehicle was stopped at a traffic light when the vehicle was hit on the left side by another vehicle traveling at speeds exceeding 60 mph (97 kmh). The client reports slight tenderness and achiness on (L) side of thorax and body. The significant assessment findings include: weak and thready pulse; diffuse abdominal pain, tenderness and guarding present upon palpation; skin is diaphoretic and extremities cool to touch, capillary refill +4 in extremities, and bruising is present in the (L) flank area and progresses towards the abdomen. Vital signs are temperature- 97.2° F (36.2° C), pulse- 110 beats/minute, respirations- 22 breaths/minute, blood pressure 84/46 mmHg, MAP- (57), and pulse oximetry 90% on 2 lpm O2 via nasal cannula.Which potential injuries should the triage nurse assess? (Select all that apply.) Flailed ribs. Fractured liver. Ruptured spleen. Cardiac tamponade. Tension pneumothorax.

Fractured liver. Correct Ruptured spleen. Correct The assessment priorities are based on the report of the mechanism of injury which indicated that the majority of point of impact from the motor vehicle collision was on the client's left side of the body. Along with the physical assessment and vital signs findings the client is displaying signs and symptoms indicating blunt trauma to the liver and the spleen.

The nurse assists the healthcare provider with the insertion of a pulmonary artery (PA)catheter for a client presenting with cardiogenic shock. Which action is most important for the nurse to take to prevent life-threatening complications from pulmonary artery monitoring? Fast flush the PA distal port for no more than 2 seconds. Avoid infusing blood products through the PA catheter. Clear pressure tubing of any blood afterwithdrawing a sample. Maintain 300 mmHg pressurearound thebagattached to the tubing.

Fast flush the PA distal port for no more than 2 seconds. Correct Verifying the correct placement of the pulmonary artery (PA)catheter is performed by the fast-flush square waveform test, also referred to the dynamic frequency response test. This test should be performed once a shift, if air bubbles, clots, or tubing is kinked the wave form will appear dampen or flat

Two days after surgical repair of an aortic abdominal aneurysm (AAA), the cardiac monitor is displaying sinus bradycardia and blood pressure 82/40 mmHg. Nitroprusside is infusing at 10 mcg/kg/min and 0.9% sodium chloride at 100 mL/hr. Which intervention should the nurse implement first? Place the client's head flat and elevate the feet. Stop the nitroprusside infusion. Rapidly administer 1 liter intravenous fluids. Increase the 0.9% sodium chloride to 150 mL/hr.

Stop the nitroprusside infusion. Correct The incisional anastomosis is leaking, causing a critical drop in blood pressure and bradycardia. The medication used to control the client's blood pressure, nitroprusside, should be stopped immediately. This client will then need volume and will need to be quickly returned to surgery for repair.

The nurse is caring for a client with severe burns 36 hours after the client's admission and identifies the client's potassium level of 6.0 mEq/L (6.0 mmol/L). Which drink will the nurse recommend be included in the client's diet? Milk Tea Orange juice Tomato juice

Tea Rationale The client is hyperkalemic, and potassium intake should be limited; tea is very low in potassium. Milk, orange juice, and tomato juice are all high-potassium foods and should be avoided.

Which statement best describes the condition of a client who the triage nurse assigned a red tag? The client is expected and allowed to die. The client has an immediate threat to life. The client has major injuries that require treatment. The client has minor injuries that do not require immediate treatment. Rationale A red tag indicates that the client has an immediate threat to life. A black tag indicates that the client is expected and allowed to die. The yellow tag indicates that the client has major injuries that require treatment. The green tag indicates that the client has minor injuries that do not require immediate treatment.

The client has an immediate threat to life. Rationale A red tag indicates that the client has an immediate threat to life. A black tag indicates that the client is expected and allowed to die. The yellow tag indicates that the client has major injuries that require treatment. The green tag indicates that the client has minor injuries that do not require immediate treatment.

A woman who was discharged recently from the hospital after undergoing a hysterectomy calls the clinic and states that she has tenderness, redness, and swelling in her right calf. Which priority action would the nurse instruct the client to take? "Stay in bed for at least 3 days." "Keep the legs elevated while sitting."" Apply a warm compress to the affected calf twice a day." "Go to the emergency department immediately."

"Go to the emergency department immediately." Rationale The client's description of her problem is indicative of thrombophlebitis; this is a medical emergency because it may precipitate a pulmonary embolism. The client must be assessed by a primary health care provider. Intravenous anticoagulants will probably be necessary. Although bed rest may be prescribed eventually, a delay in pharmacological treatment may jeopardize the client's status. Elevation of the legs may be prescribed eventually, after the thrombophlebitis is resolved. Although warm compresses are commonly prescribed, a delay in pharmacological treatment may jeopardize the client's status.

A client who has experienced trauma is admitted to the intensive care unit (ICU). The nurse's initial assessment findings include a Glasgow Coma Scale score of (3), pupils fixed and dilated with an absence of corneal reflex, blood pressure of 80/30 mmHg, core temperature of 95.7°F (35.4° C). The client's spouse asks the nurse when the client will wake up. How should the nurse respond? "Your spouse's condition indicates irreversible damage." "Let me contact the health care provider to answer your questions." "Each person is different and we need to wait and see what happens." "I need to initiate the volume expanders and warming blanket to stimulate a response."

"Let me contact the health care provider to answer your questions." Correct The client's neurological assessment findings of a low Glasgow score of 3, absence of pupil and corneal response, hypothermia, accompanied by hypotensive and low mean arterial pressure is indicative of a clinical brain death. The client's expected outcome is irreversible brain death and the nurse should contact the healthcare provider to talk and inform the spouse of the client's prognosis. It is not within the nurse's scope of practice to diagnose and inform the client's spouse of the clinical brain death diagnosis.

A client receives an autograft for a severe burn and is taught how to change the dressing. One week after receiving the graft, the client identifies that the edges of the graft are curling up and asks the nurse about it. Which is the best response by the nurse? "May I take a look at it?" "It's time for another graft." "Is there any sign of redness?" "It is supposed to curl up at the edges."

"May I take a look at it?" Rationale An autograft is a permanent graft that should not be rejected; the nurse would assess the site immediately. An autograft should not need to be replaced, and the edges should not curl. The nurse needs to assess the site; the responsibility of assessment should not be left to the client.

After experiencing acute coronary syndrome, a client who has begun an exercise program asks the nurse how to tell whether the exercise is helping. Which response would the nurse make? "Intermittent claudication will be reduced." "Your breathing will become regular and shallow." "Perspiration will be less when you run, and you'll use less energy." "You will be able to walk progressively longer distances before tiring."

"You will be able to walk progressively longer distances before tiring." Rationale The ability to endure progressive activity indicates that the heart is adapting to an increased workload. Intermittent claudication is related to peripheral arterial occlusive disease, not cardiopulmonary function. Breathing when exercising should be regular and deep to meet the oxygen demands of the body. Perspiration is an

As a surface rewarming measure for a client who is hypothermic, which is the highest temperature up to which the fluid can be heated for peritoneal lavage? 98.6°F (37°C) 111.2°F (44°C) 113°F (45°C) 120.2°F (49°C)

113°F (45°C) Rationale Peritoneal lavage can be performed with fluids warmed up to 113°F (45°C) as a surface rewarming measure. Warmed intravenous fluids can be used for surface rewarming; the fluids can be warmed up to 98.6°F (37°C). Heated humidified oxygen can be used for surface rewarming; the oxygen should be heated up to 111.2°F (44°C). Peritoneal lavage should not be performed with fluids warmed up to as high as 120.2°F (49°C).

Which is the minimum length of time victims of a large scale near- drowning incident brought to the emergency department would be kept under observation in the hospital? 12 hours 18 hours 23 hours 48 hours

23 hours Rationale Complications can develop in victims who are essentially free of symptoms immediately after a near-drowning episode. This secondary drowning refers to delayed death from drowning due to pulmonary complications. Consequently, all victims of near-drowning should be observed in a hospital for a minimum of 23 hours, and longer if they have comorbidities. Twelve hours and 18 hours are too soon to rule out secondary drowning. The clients need not be kept under observation for 48 hours if they do not have comorbidities.

A client is burned on the anterior part of both legs, from the knees to the feet. Which percentage of total body surface area (TBSA) burned would the nurse document in the client's hospital record, using the rule of nines? 9% 18% 27% 36%

9% Rationale The anterior surface of each lower limb is 4.5%, therefore 4.5 + 4.5 = 9. A calculation of 18%, 27%, or 36% is too large for only the anterior lower extremities.

Which condition is triaged as urgent among these client conditions? A - DISPLACED OR MULTIPLE FRACTURES B - SKIN RASH C - STRAINS AND SPRAINS D - SIMPLE FRACTURE

A - DISPLACED OR MULTIPLE FRACTURES Rationale Client A with displaced or multiple fractures does not have an immediately life-threatening condition, but needs immediate treatment when compared with other clients, so is considered urgent and triaged under urgent. Client B with a skin rash, client C with strains and sprains, and client D with a simple fracture could wait several hours and are triaged as non-urgent.

Which client would be triaged first based on condition? A - SEVERE RESP DISTRESS B - CHEST PAIN FROM TRAUMA C - HIP FRACTURE IN OLDER PX D - CYSTITIS

A - SEVERE RESP DISTRESS Rationale Client A with severe respiratory distress is triaged under emergency severity index 1 (ESI-1) and should be seen immediately because his or her condition is most severe. Client B with chest pain resulting from trauma is triaged under ESI-2 and is seen within 1 hour. Client C with a hip fracture could be delayed treatment because the condition is less severe and is prioritized as ESI-3. Client D with cystitis is triaged as ESI-4, and the client could receive delayed treatment

When helping a triage officer evaluate the victims of a large-scale disaster, which client would the nurse anticipate will be given a black tag? A client with a contusion A client with airway obstruction A client with open fractures and a distal pulse A client with extensive full-thickness body burns

A client with extensive full-thickness body burns Rationale A black-tagged client in a mass casualty triage situation is a critically ill client who is expected and allowed to die or is not treated until others have received care. This is done so that limited resources can be dedicated to saving the most lives, and not expended to save one life at the possible expense of many others. The client with extensive full- thickness body burns falls in this category. A client with a contusion would be green- tagged because a contusion is a minor injury that can be managed in a delayed fashion. A client with airway obstruction would be red-tagged. This client has an immediate threat to life and requires immediate attention. A client with open fractures with a distal pulse would be yellow-tagged. This is major injury that requires treatment within 30 minutes to 2 hours.

A male client who experienced a myocardial infarction (MI) asks the nurse what could have caused the MI, since he had been following a lifestyle of regular exercise and healthy food choices. Which response should the nurse provide to the client? A family history of heart disease is a risk factor for MI. Foods consumed when younger can cause plaque formation. Immediate medical treatment was a primary factor is survival. Myocardial tissue after a minor MI can heal with no long-term effects.

A family history of heart disease is a risk factor for MI. Correct The best response to the client's question about the causes of myocardial infarction is to explain non-modifiable risk factors for heart disease, which include genetics, age, and gender

A client reports left-sided chest pain after playing racquetball. The client is hospitalized and diagnosed with left pneumothorax. When assessing the client's left chest area, the nurse expects to identify which finding? Dull sound on percussion Vocal fremitus on palpation Rales with rhonchi on auscultation Absence of breath sounds on auscultation

Absence of breath sounds on auscultation Rationale The left lung is collapsed; therefore there are no breath sounds. A tympanic, not a dull, sound will be heard with a pneumothorax. There is no vocal fremitus because there is no airflow into the left lung as a result of the pneumothorax. Rales with rhonchi will not be heard because there is no airflow into the left lung as a result of the pneumothorax.

A pregnant client is admitted with abdominal pain and heavy vaginal bleeding. Which is the immediate nursing action? Administering oxygen Elevating the head of the bed Drawing blood for a hematocrit level Giving an intramuscular analgesic

Administering oxygen Rationale Abdominal pain and heavy vaginal bleeding indicate significant blood loss. To compensate for decreased cardiac output, oxygen is given to maintain the well-being of both mother and fetus. Elevating the head of the bed will decrease blood flow to vital centers in the brain. Drawing blood for a hematocrit level is not the priority. Giving an intramuscular analgesic may mask abdominal pain and sedate an already compromised fetus; also, it requires a primary health care provider's prescription.

A client develops bacterial meningitis. Which action is the priority nursing care? Monitoring for signs of intracranial pressure Adding pads to the side of the bed Administering prescribed antibiotics Administering glucocorticoids

Administering prescribed antibiotics Rationale For bacterial meningitis, the client's greatest need is a regimen of antibiotics to which the causative agent is sensitive. Bacterial meningitis causes increased intracranial pressure and it is important for the nurse to monitor for manifestations of increased intracranial pressure; however, in this circumstance, it is not the priority because monitoring alone does not affect outcome . Because of the risk for seizures in bacterial meningitis, padded side rails are an important nursing intervention; however, this intervention does not have priority over instituting the appropriate antibiotic therapy to eradicate the cause of the meningitis. Administration of glucocorticoids is important to improve outcomes, but antibiotic therapy is even more important because without these, the infection will continue and can be life-threatening.

Which condition will be given the highest priority for a client admitted in the emergency department who has airway obstruction, chest wall trauma, external hemorrhage, and hypoglycemia? Hypoglycemia Chest wall trauma Airway obstruction External hemorrhage

Airway obstruction Rationale The highest priority intervention is to establish a patent airway because inadequate oxygen supply to the brain may cause brain death. Assessing the metabolic conditions is done after the airway is cleared. Once the airway is cleared, then the chest wall of the client is assessed. Hemorrhage is assessed after the airway of the client is cleared.

A client in her 36th week of gestation is admitted with vaginal bleeding, severe abdominal pain, a rigid fundus, and signs of impending shock. For which intervention would the nurse prepare? A high-forceps birth An immediate cesarean birth Insertion of an internal fetal monitor Administration of an oxytocin infusion

An immediate cesarean birth Rationale The client's signs and symptoms are those of complete placental separation (abruptio placentae) for which an immediate cesarean birth is the ideal treatment. High-forceps birth is rarely used, because the forceps may further complicate the situation by tearing the cervix. The risk for fetal and maternal mortality is too high to delay action, so a fetal monitor would not be inserted. Administration of oxytocin would greatly increase the risk of fetal death.

Which activity is performed first when a client is bitten by a brown recluse spider? Removing the necrotic tissue Elevating the affected extremity Allowing complete rest to the client Applying cold compresses over the site

Applying cold compresses over the site Rationale The basic first aid for a brown recluse spider bite is to apply cold compresses over the site intermittently until there is no further progression of the wound. Cold helps decrease the enzyme activity of the venom and may limit tissue swelling and necrosis. Necrotic tissue is removed after evaluating the wound. Elevating the affected extremity is recommended, but it is not the first action. The client is allowed to rest after ensuring the client is safe.

Which information would the nurse include in a teaching plan for a client whose burns are being treated with the exposure (open) method? Aseptic techniques are required. Plants, but not flowers, are allowed. Equipment will be shared with others. Dressings will be changed every 3 days.

Aseptic techniques are required. Rationale Medical asepsis and surgical asepsis are essential for prevention of infection with the exposure method, because only antimicrobials are used with no dressing. Equipment is not shared on the burn unit. Dressings are not used with the exposure method. Neither plants nor flowers can be present.

Which client would be triaged under emergency severity index (ESI)-1 based on threat to life and stability of vital functions? A - CHEST PAIN FROM ISCHEMIA B - CARDIAC ARREST C - SIMPLE LACERATION D - HIP FRACTURE

B - CARDIAC ARREST Rationale Client B in cardiac arrest will have unstable vital functions of the ABCs (airway-breathing- circulation) and a threat to life exists. The client should be triaged under the ESI-1 level and should be seen immediately by the primary health care provider. Client A with chest pain has threatened vital functions but is not unstable and should be seen within 1 hour. This type of client is categorized as ESI-2. Client C who has a simple laceration is categorized as ESI-4, which indicates that the vital functions are stable and the client has no immediate threat to life. Client D who has a hip fracture is triaged as ESI-3, which indicates that the vital function is stable and a threat to life is unlikely.

Which biological agent of terrorism is treated with antitoxin? Plague Anthrax Botulism Smallpox

Botulism Rationale Botulism is treated with antitoxin, though several vaccines are being studied. Plague and anthrax can be treated effectively with antibiotics if sufficient supplies are available and the organisms are not resistant. Smallpox can be prevented or the incidence reduced by vaccination, even when first given after exposure.

Which laboratory test results should the nurse review for a client with this ECG tracing? Basal metabolic panel (BMP), serum Magnesium, and serum Phosphorus. Lactic Acid Cardiac enzymes and Troponins. Complete blood count.

Basal metabolic panel (BMP), serum Magnesium, and serum Phosphorus. Correct The client is experiencing unifocal premature ventricular contractions (PVCs) that can be caused by an electrolyte imbalances. The nurse should review the client's serum electrolytes such as sodium, potassium, calcium, phosphorous, and magnesium. Abnormal levels of these electrolytes can cause abnormal cardiac rhythms. Client's with hypokalemia and hypomagnesium are at an increased risk for PVCs, along with the absence of known cardiac disease, hypophosphatemia can cause ventricular contractions abnormalities.

A client returns to the intensive care unit (ICU) after having a permanent pacemaker inserted. Which finding should the nurse observe for during the immediate hours after insertion? Beck's triad. Burns around the site Hypothermia Cardiac arrhythmia.

Beck's triad. Correct Pacemakers and implantable defibrillators both require wires to be placed into the heart muscle and can cause bleeding into the pericardial sac which causes a high risk for cardiac tamponade. Signs of Beck's triad include low arterial blood pressure, distended neck veins, and distant, muffled heart sounds and are indicative of cardiac tamponade.

Which statement about the injuries caused by explosive devices used as agents of terrorism would the nurse know to be true? Penetrating injuries primarily damage the middle ear. Blast injuries primarily damage the gastrointestinal (GI) tract. Penetrating injuries are caused by the supersonic over pressurization shock wave. Blast injuries are caused by explosions in confined spaces causing structural collapse.

Blast injuries primarily damage the gastrointestinal (GI) tract. Rationale The shock wave generated by an explosion primarily causes blast injuries to the lungs, GI tract, and middle ear. Penetrating injuries are caused when shrapnel from the explosive device penetrates the skin; damage to the middle ear is not typical of penetrating injuries. Blast injuries result from the supersonic overpressurization shock wave caused by the explosion. Crush injuries, such as blunt trauma, often result from explosions in confined spaces causing structural collapse.

A client's cardiac rhythm reveals peaked "T" waves, a widening "QRS" complex and the flattening of "P" waves. Which medication should the nurse administer? Phosphate IV push. Furosemide IV push. Calcium gluconate IV push. Diluted potassium IV push.

Calcium gluconate IV push. Correct The client's cardiac rhythm is indicative of a potassium level is greater than 7.0 mEq/L. Hyperkalemia with cardiac rhythm changes should be treated with calcium gluconate. The calcium gluconate will cause an immediate effect on the electrical activity of the heart and the cardiac rhythm tracing will reflect that immediately and lower the risk of the client's cardiac

The nurse is caring for a client who is recently extubated in the post anesthesia care unit (PACU). The client has humidified oxygen per mask and suddenly develops stridor and respiratory difficulty. Which action should the nurse implement? Call a rapid response team for emergency airway management. Encourage the client to take deep breaths,cough, and expectorate. Increase the flow rate of the humidified oxygen. Suction the client's mouth and oropharynx thoroughly.

Call a rapid response team for emergency airway management. Correct Stridor post-extubation is a medical emergency. The nurse should immediately call the rapid response team for airway management.

The nurse is caring for a client in the intensive care unit (ICU) with type 1 diabetes mellitus who has a blood glucose level of 600 mg/dL (33.3 mmol/L). Which clinical manifestation is most important for the nurse to report to the healthcare provider if the blood sugar continues to rise? Change in level of consciousness. Increase in urinary output. Onset of Kussmaul respirations. Decrease in serum potassium level.

Change in level of consciousness. Correct As blood sugar rises (norm 70 to 110 mg/dl or 3.9-6.1 mmol/L SI), a client with hyperglycemia becomes dehydrated due to excessive urine output that causes a drop in blood volume and cerebral hypoperfusion. A change in the client's level of consciousness should be reported to the healthcare provider immediately.

A client is trying to talk with an endotracheal tube in place for mechanical ventilation. Which method is most readily available for the nurse to provide the client for communication? Communication board or paper pencil to write messages. Hand gestures that explain what the client wants to say. Communication by interrupting ventilator for short periods of time. Touch screen figures or text to voice communication computers.

Communication board or paper pencil to write messages. Correct A communication strategy for a client on a ventilator in ICU should be brief, minimally fatiguing, immediately available for use by client and staff and does not compromise the client's respiratory management. Although augmentative alternative communication (AAC) electronic devices are options, the most readily available tool is a communication board, paper pencil, or a letter chart to spell words and messages.

Which health care professional serves as the liaison between hospital administration and the media to release only appropriate and accurate information in the event of a mass casualty event or disaster? Triage officer Community relations officer Medical command physician Hospital incident commander

Community relations officer Rationale Mass casualty incidents tend to attract a large amount of media attention. The community relations or public information officer can serve as the liaison between hospital administration and the media to release only appropriate and accurate information. The triage officer is a primary health care provider or nurse who rapidly evaluates each client to determine priorities for treatment. The medical command physician decides the number, acuity, and resource needs of clients arriving from the incident scene to the hospital and organizing the emergency health care team response to the injured or ill clients. The hospital incident commander is a primary health care provider or administrator who assumes overall leadership for implementing the emergency plan.

Which is the function of the medical command physician according to the Emergency Preparedness and Response Plan? Deciding the number, acuity, and resource needs of clients Serving as a liaison between the health care facility and the media Assuming overall leadership for implementing the emergency plan Evaluating each client rapidly to determine priorities for treatment

Deciding the number, acuity, and resource needs of clients Rationale The function of the medical command physician, according to the Emergency Preparedness and Preparedness Plan, is deciding the number, acuity, and resource needs of clients. The function of the community relations or public information officer is serving as a liaison between the health care facility and the media. The function of the hospital incident commander is assuming overall leadership for implementing the emergency plan. The function of the triage officer is evaluating each client rapidly to determine priorities for treatment.

The paramedics bring in a client who is a victim of a high speed motor vehicle collision. The client is semi-coherent, fading in and out of consciousness. Two large bore intravenous catheters have been put in place, and one is infusing with normal saline at 100 mL/hr. Which information is the most important for the triage nurse to obtain from the paramedics? The victim's vital signs upon arrival on the scene. Status of the other victims involved in the accident. Description of the motor vehicle collision circumstances. Police custody status of the victim pending further investigation.

Description of the motor vehicle collision circumstances. Correct It is important for the nurse to obtain the "mechanism of injury". This refers to a description of what happened during the accident, such as where was the victim sitting in the vehicle, was the victim restrained or unrestrained by a seat belt, approximate speed of each vehicle during the impact, and what part of the victim's vehicle was hit. Obtaining this information will help the triage nurse in performing the assessment and looking for potential injuries as they relate to the mechanism of injury.

Which finding will the nurse expect when analyzing the cardiac rhythm for a client with first degree atrioventricular (AV) block? Every P wave is conducted to the ventricles. Some P waves are conducted to the ventricles. There are no P waves visible on the rhythm strip. None of the P waves are conducted to the ventricles.

Every P wave is conducted to the ventricles. Rationale In first degree AV block, a P wave precedes every QRS complex, but the PR interval is prolonged. Second degree heart block refers to AV conduction that is intermittently blocked. Some P waves are conducted and some are not. When no P waves are visible, rhythms such as junctional dysrhythmias or atrial fibrillation are suspected. Third degree block is often called complete heart block because no atrial impulses are conducted through the AV node to the ventricles.

The nurse is caring for a client admitted to the surgical intensive care unit (SICU) on the first postoperative day after a kidney transplantation. Which intervention should the nurse include in the plan of care to prevent hypovolemia? Give IV fluids on a 1:1 ratio from output. Administer loop diuretics. Increase sodium intake. Provide sports drinks for hydration.

Give IV fluids on a 1:1 ratio from output. Correct The transplanted kidney will produce large amounts of urine if the transplanted organ works well, which means the client is at risk for hypovolemia. The plan of care should include protocol prescriptions to replace fluid volume at a 1:1 ratio based on output.

Which is the most appropriate nursing intervention for a client admitted to the high-risk prenatal unit at 35 weeks' gestation with a diagnosis of complete placenta previa? Applying a pad to the perineal area Having oxygen available at the bedside Allowing bathroom privileges with assistanceE ducating the client regarding the intensive care nursery

Having oxygen available at the bedside Rationale If hemorrhage should occur, oxygen is necessary to prevent maternal and fetal compromise. A perineal pad is not necessary; close monitoring is required. The client admitted with a complete placenta previa is usually on complete bed rest. It is too soon to discuss the neonatal intensive care unit, because this may ultimately be unnecessary.

The triage nurse is assessing a victim of a stab wound. According to the paramedics, the victim was stabbed by a male attacker. The knife blade is 6 inches (7.2 cm) in length and 1 inch (2.5 cm) in width. The "point of entry" of the stab wound is two inches above and left of the victim's xiphoid process. Which vital organ(s) are at an increased risk of being in the direct path of injury? (Select all that apply.) Heart. Lungs. Stomach. Intestines. Diaphragm.

Heart. Lungs. Diaphragm. Stab wounds are low velocity wounds that the path of injury to the underlying organs is determined by the direction of the path of the impaled knife and the length and width of the blade. The gender of the attacker is important to know because females tend to stab in a downward direction (trajectory) and males tend to stab in an upward direction (trajectory).

The nurse is caring for a client who is scheduled to have a pigskin graft applied to a burned area. About which type of graft will the nurse educate the client? Isograft Allograft Homograft Heterograft

Heterograft Rationale A heterograft (xenograft) involves the grafting of tissues from a different species. An isograft is when the donor and recipient are genetically identical (identical twins). An allograft (homograft) is from a nonidentical donor of the same species.

The nurse is caring for a client in the critical care unit who is experiencing end-stage chronic obstructive pulmonary disease (COPD). The client is receiving oxygen at 40 L/minute via Vapotherm. The healthcare provider informs the client and family that there is no further treatment available for the COPD. Which intervention should the nurse recommend that is most beneficial to the client and family? Hospice services. Intubation with mechanical ventilation. Organ donation. Home health care.

Hospice services. Correct When the healthcare provider determines that there is no further treatment available for a client with end-stage chronic obstructive pulmonary disease (COPD), hospice services are often the most beneficial to the client and family. Vapotherm is an assisted respiratory device that delivers high flow and high velocity oxygen via nasal cannula to minimizes dead space in the lungs and decreases the work of breathing for the client.

Which action would the nurse initiate first to ensure the physical safety of a client with severe preeclampsia? Institute seizure precautions. Decrease environmental stimuli. Administer the prescribed sedatives. Strictly monitor her intake and output.

Institute seizure precautions. Rationale This client may become eclamptic suddenly and have a seizure; seizure precautions are necessary to keep her from injuring herself and the fetus. Decreasing environmental stimuli is important; however, the client's safety should be ensured first. Administering sedatives will help reduce nervous system irritability; it will not ensure safety if the client has a seizure. Monitoring intake and output will be required when magnesium sulfate therapy is started.

In a mass casualty situation, which is the facility-level organizational model for disaster management used by the hospital or long-term care facility? Emergency operations center (EOC) Hospital Incident Command System (HICS) National Incident Management System (NIMS) Federal Emergency Management Agency (FEMA)

Hospital Incident Command System (HICS) Rationale The facility-level organizational model for disaster management is the HICS. It attempts to standardize disaster operations by formally structuring roles under the hospital or long- term care facility incident commander with clear lines of authority and accountability for specific resources. The EOC, NIMS, and FEMA are not facility-level organizational models for disaster management. The EOC is established by the HICS personnel in a designated location with accessible communication technology to manage the overall incident. The NIMS is the overall system for incident management of which the HICS is a part; it is implemented by the Department of Homeland Security and FEMA. The FEMA provides numerous online resources so that people are better prepared for disasters and are able to respond more self-sufficiently to incidents and hazard situations in their own communities.

In a mass casualty situation, which health care professional can direct departments such as physical therapy or a surgical clinic to cancel their usual operations to convert the space into a minor treatment area? Triage officer Community relations officer Medical command physician Hospital incident commander

Hospital incident commander Rationale The hospital incident commander's role is to take a global view of the entire situation and facilitate client movement through the system in a mass casualty situation. He or she could direct departments such as physical therapy or a surgical clinic to cancel their usual operations to convert the space into a minor treatment area. The triage officer is a primary health care provider or nurse who rapidly evaluates each client to determine priorities for treatment. The community relations or public information officer can serve as the liaison between hospital administration and the media to release only appropriate and accurate information. The medical command physician decides the number, acuity, and resource needs of clients arriving from the incident scene to the hospital and organizing the emergency health care team response to the injured or ill clients.

The nurse is preparing to suction an intubated client's endotracheal tube (ETT). Which action should the nurse do first prior to suctioning the client's ETT? Instill sterile normal saline into the ETT. Hyperoxygenate with 100% FiO 2 for 30 seconds. Increase the respiratory rate settings on the ventilator. Adjust the suction apparatus to low intermittent setting.

Hyperoxygenate with 100% FiO 2 for 30 seconds. Correct Prior to suctioning an ventilated client's endotracheal tube (ETT), the nurse should hyperoxygenate the client with 100% FiO2 . Do not routinely place sterile normal saline into the ETT prior to suctioning, unless indicated to loosen up mucus plugs or secretions. Studies have shown the use of routine instillation of normal saline prior to suctioning can cause oxygen desaturation and backwash potentially infectious agents deeper into the lungs, increased intracranial pressure, and cause excessive coughing and bronchospasm

Which type of shock would the nurse monitor for in a client with a ruptured abdominal aortic aneurysm? Obstructive Neurogenic Cardiogenic Hypovolemic

Hypovolemic Rationale Hypovolemic shock occurs because of blood loss from the circulation when an abdominal aneurysm ruptures. Obstructive shock occurs from physical obstruction impeding the filling or outflow of blood, such as cardiac tamponade or pulmonary embolism. Neurogenic shock results from spinal cord or head injury, which cause vasodilation due to loss of sympathetic nervous system vasoconstrictor tone. Cardiogenic shock results from a decrease in cardiac output.

For which complication would the nurse closely monitor a client with a diagnosis of abruptio placentae? Cerebral hemorrhage Pulmonary edema Impending seizures Hypovolemic shock

Hypovolemic shock Rationale With abruptio placentae, uterine bleeding can result in massive internal hemorrhage, causing hypovolemic shock. A cerebral hemorrhage may occur with a dangerously high blood pressure; there is no information indicating the presence of a dangerously high blood pressure. Pulmonary edema may occur with severe preeclampsia or heart disease, and seizures are associated with severe preeclampsia; there is no information indicating the presence of these conditions.

Which action would the nurse take before developing the teaching plan for a client who has had a myocardial infarction? Identify the learning needs of the client. Determine the nursing goals for the client. Explore the use of group teaching for the client. Evaluate the community resources available to the client.

Identify the learning needs of the client. Rationale For teaching to be meaningful, the client must have a need to learn and a readiness to learn. These factors need to be identified before a teaching plan is formulated. Determining the nursing goals for the client eliminates the client from the goal-setting process; active participation by the client increases motivation and retention. Exploring the use of group teaching for the client is not the initial step; learning needs must be determined first to see whether group learning is appropriate; also, group learning must be available as an option. Evaluating community resources is not the initial step; assessment of learning needs comes first.

Which intervention is the highest priority prehospital intervention for a client bitten by a snake? Sucking the venom out of the bitten area Offering alcohol to the client Applying ice to the affected area Immobilizing the affected extremity using a splint

Immobilizing the affected extremity using a splint Rationale The priority nursing intervention for a client bitten by a snake is to immobilize the affected extremity using a splint because this may limit the spread of venom. The affected area should not be sucked because it will further harm the client. Alcohol should not be offered because it can cause the venom to spread through vasodilation. Applying ice to the affected area is ineffective and may worsen the client's condition.

The nurse is caring for a client with severe sepsis related to a ruptured appendix.The clientis diaphoretic and reports lower extremity spasms. The nurse observes respirations that are uneven and labored. Arterial blood gas (ABG) results are pH 7.60, PaCO2 25 mmHg, HCO3 24 mEq/L, and PaO2 24 mmHg. Which assessment finding warrants immediate intervention by the nurse? Increased pulmonary secretions. Intercostal muscle retraction. Decreased breath sounds. Bronchovesicular breath sounds.

Intercostal muscle retraction. Correct Intercostal muscle retraction is a critical sign of respiratory muscle fatigue that is likely to lead to acute respiratory failure, requiringintubation withmechanical ventilation. The ABG results reveal respiratory alkalosis as evidenced by an increased pH and decreased PaCO2 with a normal HCO3.

The nurse receivesreport for a client with a history of heroin and alcohol abuse who has right arm cellulitis from a puncture wound. 0.9% sodium chloride is infusing at 50 mL/hr and oxygen at 2 liters per nasal canula. The client is flushed, diaphoretic, and slow to respond to verbal stimuli. Vital signs include oxygen saturation 94%, temperature 101° F (38.3° C), heart rate 124 beats/minute, respirations 26 breaths/minute, and blood pressure 88/24 mmHg. Which intervention should the nurse implement first? Administer antipyretic suppository. Obtain 2 sets of blood cultures. Increase IV fluids to 150 mL/min. Monitor client for withdrawal signs.

Increase IV fluids to 150 mL/min. Correct The client is presenting with signs of sepsis. Fluid resuscitation is the initial treatment for hypotension in clients with septic shock. Vital signs of an elevated temperature above 100.5 F (38.1 C), tachycardia and tachypnea, accompanied with a low blood pressure can be indicative of a client who is becoming septic and going into shock.

Initially after a stroke, the client's pupils are equal and reactive to light. Four hours later, the nurse identifies that one pupil reacts more slowly than the other and the client's systolic blood pressure is increasing. For which condition would the nurse prepare to intervene? Spinal shock Brain herniation Hypovolemic shock Increased intracranial pressure

Increased intracranial pressure Rationale Increased intracranial pressure is manifested by a sluggish pupillary reaction and elevation of the systolic blood pressure. Spinal shock is manifested by a decreased systolic blood pressure with no pupillary changes. Brain herniation is manifested by dilated pupils and severe posturing. Hypovolemic shock is indicated by a decrease in systolic pressure and tachycardia, with no changes in pupillary reaction.

A client reports shortness of breath and chest pressure radiating down the left arm. The client is receiving 2 liters of oxygen via nasal cannula and has two saline lock intravenous catheters. The nurse performs a 12 lead electrocardiogram (ECG) that shows ST segment elevation in leads II, III, aVF, and V4R. Which action should the nurse implement first? Give0.3 mg nitroglycerin sublingual. Administer4 mgIV morphine sulfate. Measure the ST segment height. Infuse 0.9% sodium chloridebolus.

Infuse 0.9% sodium chloridebolus. Correct This 12 lead electrocardiogram (ECG) myocardial injury is in the inferior and right ventricular portion of the heart muscle and requires an intravenous (IV) fluid bolus to fill the right ventricle for the heart to pump.Medications such as morphine sulfate and nitroglycerin will cause profound hypotension due to vasodilation if fluids are not given first.

A client's blood pressure drops suddenly from 160/90 mmHg to 60/40 mmHg minutes after the nurse administers a 0.3 mg sublingual dose of nitroglycerin when the client reports crushing chest pain. The client is experiencing severe nausea, dizziness, and left arm numbness. Which intervention should the nurse implement? Give a PRN antiemetic medication. Administer second dose of nitroglycerin. Infuse a rapid0.9% normal saline bolus. Applyexternal pace maker pads to chest.

Infuse a rapid0.9% normal saline bolus. Correct When chest pain is treated with a vasodilator such as nitroglycerin and the blood pressure falls suddenly to a critical level, this may indicate a right ventricular infarction and requires immediate infusion of fluid to prime the right side of the heart.

The emergency department nurse is triaging a client whose spouse reports a sudden onset of symptoms to include fever, tremors, and decreased level of orientation and psychotic behavior, accompanied with the tachycardia and palpitations. The only significant medical history of the client is 4 days post-operative laparotomy appendectomy without complications and discharged to home 2 days ago. Which intervention should the nurse do first? Initiate a large bore intravenous catheter. Perform a twelve-lead electrocardiogram test. Insert an indwelling urinary catheter drainage system. Obtain serum elctrolytes and kidney function test laboratory specimens.

Initiate a large bore intravenous catheter. Correct The thyroid levels of a client experiencing a thyroid storm are elevated and often are reflective of the same elevated levels of a client experiencing hyperthyroidism. The diagnosis of a thyroid storm is based on a client's clinical presentation of symptoms. A sudden onset of very high temperature, tachycardia, palpitations, decreased level of orientation, tremors and psychotic behaviors are often signs of a thyroid storm. The thyroid storm is often brought on by stress related to a present illness, surgery, general anesthesia, or an infection. The nurse should recognize the client's clinical presentation, accompanied with the recent history of surgery. The nurse's first action should be to initiate a large bore intravenous catheter, so propranolol (Inderal) can be started to help antagonize the peripheral effects of the circulating thyroid hormone.

The nurse is caring for a client who is diagnosed withdiabetic ketoacidosis (DKA). The client reports abdominal pain and nausea, and presents with fruity-scented breath. The nurse performs a finger stick blood glucose with a reading too high to register. Which intervention is most important for the nurse to implement? Initiate a one liter bolus of 0.9% sodium chloride. Set up an IV pump to infuse IV insulin per protocol. Draw blood to evaluate a complete metabolic panel. Administer a prn IV dose of prescribed antiemetic.

Initiate a one liter bolus of 0.9% sodium chloride. Correct A client who is diagnosed with diabetic ketoacidosis and blood glucose is too high to register on a bedside glucometer, needs to receive immediate bolus of one liter of normal saline infusion, until actual blood glucose, potassium, and phosphorus levels are known from a serum laboratory screening.

The nurse is caring for a client admitted to the critical care unit with multiple traumatic injuries sustained in a motor vehicle collision. The client has a Glasgow Coma Score of 6. Which intervention should the nurse prepare for the client? Intubation with mechanical ventilation. Nasogastric tube placement. Advanced cardiac life support. 12-lead electrocardiogram (ECG).

Intubation with mechanical ventilation. Correct A Glasgow Coma Scale (GCS) is used to determine the level of consciousness of a client with traumatic brain injury. This client has multiple traumatic injuries and a GCS score of 6 may indicate the need for mechanical ventilation support. The nurse should prepare the client for intubation and mechanical ventilation.

Which information would the nurse include when explaining the purpose of a thallium scan to the client who has a history of chest pain? It monitors action of the heart valves. It assesses myocardial ischemia and perfusion. It visualizes ventricular systole and diastole. It identifies the adequacy of the conduction system.

It assesses myocardial ischemia and perfusion Rationale Thallium imaging is used to assess myocardial ischemia or necrotic muscle tissue related to angina or myocardial infarction. Necrotic or scar tissue does not extract the thallium isotope, leading to cold spots. Action of the heart valves is available from an echocardiogram or, if indicated, from a cardiac catheterization with an angiography. Visualization of the ventricular systole and diastole is determined by cardiac angiography. Identifying the adequate cardiac conduction is determined by an electrocardiogram.

A client with diabetic ketoacidosis is admitted to the intensive care unit and is manifesting respirations that are rapid and deep. Which descriptive term should the nurse use to document the client's breathing pattern? Kussmaul respirations. Cheyne stokes respirations. Apnea. Orthopnea.

Kussmaul respirations. Correct Metabolic acidosis in DKA causes compensatory responses to increase the blood pH which results in Kussmaul respiration in a effort to blow off CO2 and adjust blood pH. The nurse should document the client's respiratory rate and Kussmaul respiratory pattern.

A client who arrived in the emergency department is experiencing status epilepticus. Which medication should the nurse administer immediately? Lorazepam IV. Phenytoin PO. Morphine IV. Levetiracetim PO.

Lorazepam IV. Correct A client who is experiencing status epilepticus requires a CNS depressant to stop continuous seizures that can be life threatening. The nurse should administer lorazepam IV for rapid therapeutic action.

A client is admitted to the post-anesthesia care unit after a segmental resection of the right lower lobe of the lung. A chest tube drainage system is in place. When caring for this tube, what would the nurse do? Raise the drainage system to bed level and check its patency Clamp the tube when moving the client from the bed to a chair Mark the time and fluid level on the side of the drainage chamber Secure the chest catheter to the wound dressing with a sterile safety pin

Mark the time and fluid level on the side of the drainage chamber Rationale The fluid level and time must be marked so that the amount of drainage in the chest tube drainage system can be evaluated. The drainage system must be kept below chest level to promote drainage of the pleural space so the lungs can expand. Clamping the tube can produce backpressure, which may cause fluid to move into the pleural space from which it came, producing a tension pneumothorax. The catheter is secured by skin sutures, not to a dressing with a safety pin.

A client diagnosed with heart failure has hemodynamic monitoring in place. Which actions should the nurse perform to obtain accurate readings from the hemodynamic monitor? (Select all that apply.) Measure the pressure readings in between the client's breaths. Place the transducer at the client's atria level and pulmonary artery level. Maintain a maximum pressure of 100 mmHg for the flush line continuously. Change out the intravenous solutions infusing via central lines every 12 hours. Calibrate the hemodynamic monitor by zeroing the transducer at the start of each shift.

Measure the pressure readings in between the client's breaths. Correct Place the transducer at the client's atria level and pulmonary artery level. Correct Calibrate the hemodynamic monitor by zeroing the transducer at the start of each shift. Correct To ensure accurate hemodynamic pressure readings, the nurse should place the transducer at the client's atria level and pulmonary artery level. The nurse also should calibrate the transducer to "0" mmHg at the beginning of each shift, and measure the pressure readings in between the client's breaths.

Which emergency medical service agency offers service such as first aid stations and special-need shelters during a disaster or pandemic disease outbreak? Medical Reserve Corps (MRC) National Disaster Medical System (NDMS) Disaster Medical Assistance Team (DMAT) Federal Emergency Management Agency (FEMA)

Medical Reserve Corps (MRC) Rationale The MRC may help staff hospitals or community health settings that face shortages and provide first aid stations or special-need shelters. The NDMS manages mass fatalities, emergency animal care, and establishes fully functional field surgical facilities. A DMAT is a medical relief team deployed to a disaster area with enough medical equipment and supplies to sustain operations for 72 hours. FEMA provides Community Emergency Response Team (CERT) training so that people are better prepared for disasters and hazard situations in their own communities.

Which personnel are responsible for identifying the need for and calling in specialty trained providers in emergencies? Triage officer Public information officer Medical command physician Hospital incident commander

Medical command physician Rationale The medical command physician is responsible for identifying the resource needs of the clients. The medical command physician identifies the need for and calls in the specialty trained providers in emergencies. Triage officers are responsible for rapidly evaluating each client to determine priorities for treatment. The public information officer serves as a liaison between the health care facility and the media. The hospital incident commander is the one who assumes overall leadership for implementing the emergency plan.

A client is bitten by a snake while out hiking with the nurse. Which action would the nurse take first? Take digital photographs of the snake. Call for immediate emergency assistance. Remove jewelry and any constricting clothing. Move the client to a safe area away from the snake.

Move the client to a safe area away from the snake. Rationale The first priority for a client bitten by a snake is to move the client to a safe area away from the snake and encourage rest to decrease venom circulation. The digital photographs of the snake are taken to identify the snake after the client is moved to a safe area. Emergency assistance is called after the client is safe. The client's jewelry and constrictive clothing is removed but is not the priority.

A client in the emergency department presents with signs and symptoms indicative of an opioid overdose. Which medication would the nurse anticipate administering? Naloxone Methadone Epinephrine Amphetamine

Naloxone Rationale Naloxone is an opioid antagonist that displaces opioids from receptors in the brain, reversing respiratory depression. Methadone is a synthetic opioid that causes central nervous system depression; it will accelerate the effects of the overdose. Epinephrine will have no effect on respiratory depression stemming from of an overdose of a narcotic. Amphetamine is a stimulant, not an opioid antagonist.

The nurse is analyzing an arterial blood gas of a client who is mechanically ventilated. The ABG results are pH - 7.37; paCO2 - 30 mmHg; HCO3 - 28mEq/liter. Which should the nurse recognize as a cause of these findings? Decreased respiratory rate causing respiratory acidosis. Decreased respiratory rate causing respiratory alkalosis. Diarrhea with a fully compensated metabolic acidosis. Nasogastric suction with a fully compensated metabolic alkalosis.

Nasogastric suction with a fully compensated metabolic alkalosis. Correct The normal arterial blood gas (ABG) levels are pH: 7.35-7.45; paCO2: 35-45 mmHg; HCO3: 22-26 mEq/liter. In fully compensated respiratory acidosis the pH is within normal limits because compensation has occurred. In compensation, the opposite of the disorder compensates to bring the pH to normal range. In this case, the CO2 is decreased to compensate for the elevated HCO3. Nasogastric suction removes acid which leads to metabolic alkalosis

The client with which treatment needs belongs to class I according to the disaster triage tag system? Can wait a short time for treatment Is dead or expected to die Needs emergency treatment Has no urgent need for treatment

Needs emergency treatment Rationale Emergent clients are identified with red tags and belong to class I according to the disaster triage tag system. Clients who can wait a short time for treatment are identified by yellow tags and belong to class II according to the disaster triage tag system. Clients who are expected to die or are dead are given a black tag and belong to class IV in the disaster triage tag system. Clients who have no urgency for treatment are issued green tags and belong to class III.

The nurse is caring for a client in the critical care unit who has type 2 diabetes mellitus and is admitted with hyperglycemic hyperosmolar syndrome (HHS). The health care provider prescribes an insulin drip of 0.1 unit/kg per hour based on a current blood glucose level of 670 mg/dL (35.3 mmol/L) . Which intervention should the nurse perform during this infusion? Obtain blood glucose levels hourly. Give potassium chloride 40 mEq per secondary infusion. Infuse Dextrose 5% with 0.45% NaCl (D 5 1/2 NS). Initiate a 2,000 calorie diabetic diet.

Obtain blood glucose levels hourly. Correct Administering an insulin drip for a client with hyperglycemic hyperosmolar syndrome (HHS) should cause blood glucose levels to drop 50 to 70 mg/dL per hour. The nurse should perform hourly blood glucose monitoring to evaluate the effectiveness of the insulin drip.

The health care provider has determined that a client has irreversible brain damage with subsequent brain death. Organ donation is discussed with the family. Which action should the nurse take prior to contacting the organ procurement organization (OPO)? Obtain informed consent. Disconnect the ventilator. Remove all jewelry. Contact the medical examiner.

Obtain informed consent. Correct When brain death has been determined by the healthcare provider, the organ donation process may be initiated. The nurse should obtain signed informed consent from the family prior to contacting the organ procurement organization (OPO).

A client with syndrome of inappropriate secretion of antidiuretic hormone (SIADH) is admitted to the intensive care unit with a serum sodium level of 112 mEq/L or mmol/L. Which protocol prescription should the nurse implement first? Obtain serum sodium levels every 4 hours. Provide oral sodium chloride supplements. Monitor fluid restriction and document hourly intake and output. Initiate normal saline IV at 100 mL/hour.

Obtain serum sodium levels every 4 hours. Correct A client diagnosed with syndrome of inappropriate secretion of antidiuretic hormone (SIADH) can experience sodium levels that are dangerously below the norm range of 136 to 145 mEq/L or 136 to 145 mmol/L (SI units). The first action is to evaluate the client's serum sodium levels to determine fluid and electrolyte correction with isotonic saline based on the client's status of hypotonic hyponatremia.

The nurse is analyzing an arterial blood gas (ABG) of a client who has a nasogastric tube to low suction. The ABG results are pH - 7.48; paCO 2 - 50 mmHg; HCO 3 - 27mEq/liter. How should the nurse interpret this blood gas? Partially compensated respiratory acidosis. Partially compensated respiratory alkalosis. Partially compensated metabolic acidosis. Partially compensated metabolic alkalosis.

Partially compensated metabolic alkalosis. Correct The normal arterial blood gas (ABG) levels are pH: 7.35-7.45; paCO2: 35-45 mmHg; HCO3: 22-26 mEq/liter. In partially compensated metabolic alkalosis the pH is not within normal limits, compensation is attempting to correct the pH. In compensation, the opposite of the disorder compensates to bring the pH to normal range. In this case, the paCO2 is elevated to compensate HCO3 being elevated

A client falls off a ladder approximately 15 feet high and is admitted to the ICU for observation due to a small intracranial bleed noted in the left occipital area of the brain as observed on the CT Scan done in the emergency department. The client has been stable for the past 12 hours. The client reports to the nurse a new onset of pain in the left shoulder. Which action should the nurse do next? Contact the healthcare provider. Perform an abdominal assessment. Observe the client's pupillary response. Examine the left shoulder's range of motion.

Perform an abdominal assessment. Correct Injury to the spleen, especially an encapsulated splenic hemorrhage may be difficult to diagnose initially because the signs of bleeding do not become evident immediately. The fact the client fell approximately 15 feet resulting in a fractured skull and now reports a new onset of left shoulder pain may be indicative of a splenic rupture. The referred shoulder pain is called "Kehr's sign". The nurse should initially assess the abdomen for signs of an abdominal injury. The nurse needs to assess the abdomen for distention, guarding, rebound tenderness, and rigidity. The nurse also needs to closely monitor the client's hemodynamic status for signs of hypovolemic depletion due to the splenic hemorrhage.

Which biological agents of terrorism can be treated effectively with antibiotics? Select all that apply. One, some, or all responses may be correct. Plague Anthrax Botulism Smallpox Tularemia

Plague Anthrax Tularemia Rationale Anthrax, plague, and tularemia are treated effectively with antibiotics if sufficient supplies are available and the organisms are not resistant. Botulism is treated with antitoxin, though several vaccines are being studied. Smallpox can be prevented or the incidence reduced by vaccination, even when first given after exposure.

Which staff members will be the first to manage the care of victims from an accident at the nuclear power plant? Emergency medicine health care provider Emergency medical technicians Paramedics Prehospital care providers

Prehospital care providers Rationale Prehospital care providers are the first caregivers before clients are transported to the emergency department by ambulance or helicopter. Paramedics are the advanced life support (ALS) providers who generally perform advanced techniques. These staff members provide advanced care but are not given priority to help in providing transport. Emergency medicine health care providers are professionals with specialized education and training in emergency client management. Emergency medical technicians render basic life support (BLS) interventions to clients.

A group of bomb blast victims needs to be transported to the hospital immediately. Which staff members of an emergency unit are most appropriate to handle this situation? Triage officer Medical command physician Prehospital care providers Emergency medicine physician

Prehospital care providers Rationale Prehospital care providers are the first caregivers who provide transport to the emergency department by ambulance or helicopter for clients in an emergency situation. Triage officers are responsible for evaluating each client to determine priorities for treatment. The medical command physician decides the number, acuity, and resource needs of clients. Emergency medicine health care providers have specialized education and training in emergency client management.

Which scenarios mentioned by the registered nurse is considered secondary acute care? Prepares a client who has suffered from repeated cerebral attacks for a CT scan Performing physical examinations and monitoring fetal movement in a pregnant woman Teaching family members about the importance of being vaccinated and the risks associated with a lack of vaccinations Checking a client's heart rate and blood pressure before administering entacapone and isoproterenol concurrently

Prepares a client who has suffered from repeated cerebral attacks for a CT scan Rationale Secondary acute care includes emergency care, acute medical-surgical care, and radiological procedures for acute problems. Preparing a client for a CT scan after repeated cerebral attacks qualifies as secondary acute care. Caring for a pregnant woman by performing physical examinations and monitoring fetal movement is considered primary care (health promotion). Teaching family members about the importance of being vaccinated and the risks of missing vaccinations are examples of preventive care. Checking the client's heart rate and blood pressure before administering medication is an example of preventive care.

The nurse reports findings to the healthcare provider for a client who is admitted to the intensive care unit today with chronic obstructive pulmonary disease (COPD). When the nurse completes the report using the SBAR format, which statement best supports the nurse's reason for calling the healthcare provider? Prescription for an additional respiratory treatment. Admission today with difficulty breathing. History of COPD. Presence of expiratory wheezes in the lower lobes.

Prescription for an additional respiratory treatment. Correct The SBAR reporting format uses client information that includes the Situation, Background, Assessment, Recommendation. The nurse should complete the report with a recommendation, such as a prescription for an additional breathing treatment.

Which goal is the nurse trying to achieve by reinforcing to the client that it is important to seek treatment for primary open-angle glaucoma (POAG)? Prevent cataracts Prevent blindness Prevent retinal detachment Prevent blurred distance vision

Prevent blindness Rationale POAG progresses gradually without symptoms; if untreated, blindness occurs. Peripheral vision slowly disappears until tunnel vision occurs in which there is only a small center field. Without treatment, eventually all vision is lost. POAG is not related to the development of cataracts, retinal detachment, or blurred distance vision.

What does the nurse consider to be the priority nursing intervention for a client on diuretic therapy who has developed metabolic alkalosis? Preventing falls Monitoring electrolytes Administering antiemetics Adjusting the diuretic therapy

Preventing falls Rationale A client with alkalosis has hypotension and muscle weakness, which increases the risk for injury due to falls; therefore, to prevent injury, the priority nursing care is to prevent falls. Monitoring electrolytes daily until they return to normal is not the priority nursing care. Antiemetics are prescribed for vomiting and are given low priority. Once the client is protected from the risk for injury, diuretic therapy is adjusted.

Which action would the nurse perform as the highest priority for a client with trauma in the emergency unit? Applying dry dressing Evaluating chest expansion Providing adequate oxygen (O 2) supply. Applying direct pressure on a bleeding site

Providing adequate oxygen (O 2) supply. Rationale The nurse would prioritize care while caring for a client with trauma in the emergency department. Evaluation of chest expansion and respiratory effort, as well as evidence of chest wall trauma, helps assess breathing, a primary survey. The highest priority intervention is to establish a patent airway by providing adequate O 2 supply, thereby reducing the brain injury and progression to anoxic brain death. Direct application of pressure on the bleeding site with thick, dry dressing material helps reduce external hemorrhage.

Which inference would the nurse draw when crackles are heard while auscultating the lungs of a client admitted with severe preeclampsia? Seizure activity is imminent. Pulmonary edema may have developed. Diaphragmatic function is being impaired by the enlarged uterus. Bronchial constriction was precipitated by the stress of pregnancy. Rationale Pulmonary edema is associated with severe preeclampsia; as vasospasms worsen, capillary endothelial damage results in capillary leakage into the alveoli. Crackles are not an indication of an impending seizure; signs of an impending seizure include hyperreflexia, developing or worsening clonus, severe headache, visual disturbances, and epigastric pain. Pregnancy does not precipitate bronchial constriction, although the hormones associated with pregnancy can cause nasal congestion. Impaired diaphragmatic function is a discomfort associated with pregnancy that may result in shortness of breath or dyspnea, not crackles.

Pulmonary edema may have developed. Rationale Pulmonary edema is associated with severe preeclampsia; as vasospasms worsen, capillary endothelial damage results in capillary leakage into the alveoli. Crackles are not an indication of an impending seizure; signs of an impending seizure include hyperreflexia, developing or worsening clonus, severe headache, visual disturbances, and epigastric pain. Pregnancy does not precipitate bronchial constriction, although the hormones associated with pregnancy can cause nasal congestion. Impaired diaphragmatic function is a discomfort associated with pregnancy that may result in shortness of breath or dyspnea, not crackles.

The client with which color tag would be treated first, according to the disaster triage tagging system? Red Black Green Yellow

RED Rationale The red tag is applied to clients who require immediate treatment according to the disaster triage tag system, so client A requires immediate treatment. A black tag is applied to clients who are dead or expected to die and are not prioritized for immediate critical care. The green tags are applied to clients who can ambulate on their own, and there is no need to attend to these clients first. The yellow tag is applied to clients who can wait a short duration for treatment and who can be treated after clients with red tags are treated.

Which would the registered nurse say is the priority action of first responders after a community disaster? Providing on-site first aid Providing emergency care Removing the victims from danger Categorizing victims using triage system

Removing the victims from danger Rationale First-responders in a disaster include other disaster-trained emergency personnel who had specific search and rescue training. The priority action of first responders is to remove injured and uninjured victims. The nursing staff provides on-site first aid and emergency care. After removing victims from danger, the health care personnel categorize victims using the triage system.

The nurse is analyzing an arterial blood gas (ABG) of a client who is mechanicallly ventilated. The ABG results are pH- 7.52; paCO2- 30 mmHg; HCO3- 28 mEq/liter. How should the nurse interpret this blood gas? Respiratory acidosis. Respiratory alkalosis. Metabolic acidosis. Metabolic alkalosis.

Respiratory alkalosis. Correct The normal arterial blood gas (ABG) levels are pH: 7.35-7.45; paCO2: 35-45 mmHg; HCO3: 22-26 mEq/liter. In respiratory alkalosis, the pH and HCO3 is increased and the CO2 is decreased.

A client with which condition would the nurse treat first according to a three-tiered triage system? Respiratory distress Multiple soft tissue injuries New onset of respiratory tract infection Skin rash and a simple lower limb fracture

Respiratory distress Rationale According to a three-tiered triage system, respiratory distress is categorized as an emergent or life-threatening condition. Clients with respiratory distress should be treated first. Multiple soft tissue injuries are categorized as urgent but not life-threatening according to a three-tiered triage system. Clients with multiple soft injuries and new onset of respiratory tract infection can be treated after treating the client with respiratory depression. A skin rash and simple leg fracture are considered nonurgent conditions, and the client can wait for hours to receive treatment.

A client diagnosed with an end-stage terminal illness has decided to discontinue treatment. The client has become very detached and does not want to participate in the plan of care. Which action should the nurse implement first? Initiate a referral for a mental health consultation. Encourage the client to participate in their plan of care. Review the client's medical record for documented religious preference. Contact the hospital chaplain to provide spiritual counseling and guidance.

Review the client's medical record for documented religious preference. Correct Individuals who follow the teachings of Buddha believe that "detachment" is the way to obtain relief from suffering. The teachings of Buddha also believe dying is natural process and in reincarnation. When an individual dies, the Buddhist believes the person is transitioning into a new life.

Which statement would the nurse know to be true regarding the characteristics of sarin? Sarin causes skin burns and blisters. Sarin can cause death within minutes of exposure. A garlic-like odor and brown color are characteristics of sarin. A single dose of pralidoxime chloride (2-PAM chloride) reverses the effects of long-term sarin exposure.

Sarin can cause death within minutes of exposure. Rationale Sarin is a highly toxic nerve gas that can cause death within minutes of exposure. Mustard gas causes skin burns and blisters. A garlic-like odor and brown color are characteristics of mustard gas. Although pralidoxime chloride (2-PAM chloride) is used as an antidote for nerve agent poisoning, multiple doses may be needed to reverse the effects of nerve agents; also, sarin acts by paralyzing the respiratory muscles.

Which equipment will the nurse prepare for a client whose burn wounds are scheduled to be debrided mechanically? Enzymatic agents Scissors and forceps Autolytic semiocclusive dressing Continuous passive-motion device

Scissors and forceps Rationale Mechanical debridement means to physically remove dirt, damaged or dead tissue, and cellular debris from a wound or burn so that infection is prevented and healing is promoted. Scissors, forceps, or scalpels may be used along with hydrotherapy. Enzymatic preparations are used to debride chemically by dissolving and removing necrotic tissue. A mechanical device that continually moves an extremity is called continuous passive range of motion and is used for knee surgery. Autolytic debridement includes semi-occlusive or occlusive dressings to soften dry eschar by autolysis.

Which conditions of the clients who are admitted with injuries due to a bus accident are prioritized under the emergent classification? Select all that apply. One, some, or all responses may be correct. Shock Contusion Open fracture Airway obstruction High cervical spinal cord injury

Shock Airway obstruction Rationale Clients with shock and airway obstruction have immediate threat to life and are triaged under the emergent class, or class 1. The client with contusion is triaged under the nonurgent, or class 3, category because a contusion is a minor injury and the client is ambulatory. The client with an open fracture has a major injury and is triaged under urgent, or class 2, but not the emergent class. The client with a high cervical spinal cord injury may be classified as expectant, or class 4, which means the client is deceased or likely to die before medical intervention can be mobilized.

The parents of a preterm newborn visit the neonatal intensive care unit (NICU) for the first time. They are obviously overwhelmed by the amount of equipment and the tininess of their baby. Which is the nurse's most appropriate response to their reaction? Placing the baby in the mother's lap Showing the parents how to touch the baby Explaining the purpose of the equipment being used Discouraging the parents from staying too long on this first visit

Showing the parents how to touch the baby Rationale Parent-infant bonding follows a natural progression involving touch, which helps the parent overcome fear. During her first contact, the mother may not be ready to hold the baby on her lap, and the newborn may not be well enough to be moved. The parents are not ready for explanations about the equipment in their current state of anxiety. Discouraging the parents from staying too long on this first visit may make them feel unwelcome and set a negative tone for future visits.

The nurse is caring for a client in the critical care unit who has a pituitary tumor and subsequent diabetes insipidus (DI). Which finding indicates the need to place the client on seizure precautions? Sodium 155 mEq/L or mmol/L. Arterial pH 7.42. Calcium 9.5 mEq/L (4.75 mmol/L) Potassium 4.9 mEq/L or mmol/L.

Sodium 155 mEq/L or mmol/L. Correct Clients with diabetes insipidus (DI) experience increased urinary output due to decreased antidiuretic hormone (ADH), which may cause dehydration and high serum sodium levels (norm 136 to 145 mEq/L or 136 to 145 mmol/L (SI units). Hypernatremia places the client at risk for seizures.

Which intervention would the nurse anticipate will be provided for the newborn of a mother with a long history of diabetes? Fast-acting insulin Special high-risk care Routine newborn care Limited glucose intake

Special high-risk care Rationale The infant of a diabetic mother is a newborn at risk because of the interaction between the maternal disease and the developing fetus. The newborn of a mother with type 1 diabetes generally is hypoglycemic because of oversecretion of insulin by the newborn's hypertrophied pancreas. The newborn of a mother with type 1 diabetes is at high risk and requires intensive care. The newborn of a mother with type 1 diabetes is prone to hypoglycemia and will probably need increased glucose.

A client who is hypotensive is receiving an infusion of dopamine 10 mcg/kg/minute IV through a peripheral line. The client begins to report burning at the IV site. Which action should the nurse implement? Stop the infusion and notify the healthcare provider of the findings. Check the line for blood return and irrigate the peripheral IV catheter. Apply a cold compress to the site and continue the infusion's rate. Slow the infusion rate and add a secondary IV of 0.9% sodium chloride.

Stop the infusion and notify the healthcare provider of the findings. Correct Dopamine, a vasopressor, has significant vasoconstrictive action that can cause soft tissues necrosis if extravasation occurs. The nurse should stop the infusion and notify the healthcare provider.

Which role does the "doorkeeper" enact in a critical incident stress debriefing (CISD) team? Helping dispel any feelings of blame or guilt in the participants Encouraging group discussion through asking a series of questions Educating participants about self-care concepts and coping strategies to use immediately Talking with anyone who leaves the session early in an effort to have him or her return or accept follow-up

Talking with anyone who leaves the session early in an effort to have him or her return or accept follow-up Rationale The role of the "doorkeeper" is to keep inappropriate people like the media and spectators out and to talk with anyone who leaves the session early in an effort to have him or her return or accept follow-up. The CISD group leader helps dispel any feelings of blame or guilt in the participants, encourages group discussion through asking a series of questions, and educates participants about self-care concepts and coping strategies to use immediately.

A client in the intensive care unit receives a STAT prescription for mannitol IV for cerebral edema post closed head injury. Which action should the nurse implement when preparing to administer the medication? Use a filtered needle to draw up the medication and an in-line filter during infusion. Place atropine at bedside for use if the client has bradycardia during administration. Hyperventilate the client prior to administration to decrease intracranial pressure. Stop all sedation while mannitol is being administered per secondary infusion.

Use a filtered needle to draw up the medication and an in-line filter during infusion. Correct Mannitol is a large sugar molecule and is a crystal at room temperature. A needle filter to draw up mannitol or an in-line filter should be used for the administration of mannitol

Which problem is indicated by a positive contraction stress test (CST)? Preeclampsia Placenta previa Fetal prematurity Uteroplacental insufficiency

Uteroplacental insufficiency Rationale A CST is positive when late decelerations occur with 50% or more of contractions. A positive CST indicates uteroplacental insufficiency and a compromised fetus. Preeclampsia does not cause a positive CST result. A CST is contraindicated for a client with a suspected placenta previa, because the contractions may cause bleeding. A CST is contraindicated for a client with a suspected preterm birth or a gestation less than 33 weeks' duration, because the contractions may induce true labor.

Which clinical indicators would the nurse consider evidence of increasing intracranial pressure? Select all that apply. One, some, or all responses may be correct. Vomiting Irritability Hypotension Increased respirations Decreased level of consciousness

Vomiting Irritability Decreased level of consciousness Rationale Anorexia, nausea, and vomiting occur because of pressure on the brain. Increasing pressure on the vital centers in the brain and irritation of cerebral tissue result in irritability and seizures. Increased intracranial pressure disrupts neurons and neurotransmitters, resulting in faulty impulse transmission and an altered level of consciousness. The blood pressure will be increased, not decreased, because of pressure on the vital centers in the brain. Also, the pulse pressure increases. Pressure on the respiratory center in the medulla results in a decreased, not increased, respiratory rate. As the intracranial pressure increases, the client may exhibit Cheyne-Stokes respirations.

The nurse is preparing a client with acute kidney injury (AKI) f or hemodialysis in the intensive care unit (ICU). Which assessment should the nurse obtain prior to beginning the procedure? Weight using the ICU bed scales. Arteriovenous (AV) fistula site. Capillary refill. Urine color and clarity.

Weight using the ICU bed scales. Correct Hemodialysis filters excess fluid and toxins from the blood when renal function is severely diminished. The nurse should assess the client's weight before and after dialysis to monitor the amount of fluid removed during the procedure.

The nurse is caring for a client in the intensive care unit who is receiving mechanical ventilation due to acute respiratory failure. The family asks when the client will be extubated. Which information should the nurse provide? When the client performs spontaneous breathing in between mechanical ventilation. Once all serum electrolyte and blood chemistry levels normalize. At the completion of intravenous antibiotic therapy and the infection is resolved. When the chest x-ray shows that the inflammation is resolved.

When the client performs spontaneous breathing in between mechanical ventilation. Correct Prior to weaning, clients must demonstrate the ability to breath on their own (spontaneous breathing). The decision to wean a client from mechanical ventilation is based on the client's spontaneous respiratory rate during trials of interrupted ventilation.

A chest X-ray is prescribed for a client with possible adult respiratory distress syndrome (ARDS). Which radiographic finding represents the pathological processes of pulmonary edema and consolidation of the lungs as ARDS progresses ? White-out appearance. Infiltrates. Calcified cavities. Multiple nodules. As adult respiratory distress syndrome (ARDS) progresses, a "white-out" appearance on the chest X-ray indicates opacity of pulmonary edema and density of lung consolidation.

White-out appearance. Correct As adult respiratory distress syndrome (ARDS) progresses, a "white-out" appearance on the chest X-ray indicates opacity of pulmonary edema and density of lung consolidation.

Which assessment finding alerts the nurse to increasing intracranial pressure? Hypervigilance Constricted pupils Increased heart rate Widening pulse pressure

Widening pulse pressure Rationale Pressure on the vital centers in the brain causes an increase in the systolic blood pressure, widening the difference between the systolic and diastolic pressures. The client will be lethargic and have a lowered level of consciousness. The pupils will be unequal or dilated, not constricted. Pressure on the vital centers in the brain results in a decreased, not increased, heart rate.

Which color of cerebrospinal fluid (CSF) may indicate subarachnoid hemorrhage in the client? Hazy Yellow Brown Colorless

Yellow Rationale The yellow color of CSF can be attributed to the hemolysis of the red blood cells, which leads to increased production of bilirubin. Other causes include subarachnoid hemorrhage, jaundice, increased CSF protein, hypercarotenemia, or hemoglobinemia. Hazy or unclear CSF is indicative of an elevated white blood cell count as a result of infection. If the CSF has a brown color, it is indicative of the presence of methemoglobin, indicating a previous meningeal hemorrhage. Colorless CSF indicates a normal finding.


Set pelajaran terkait

Dictionary/Thesaurus Illuminate Quizizz/Quizlet

View Set

Vocabulary: Analyzing Accounting Transactions

View Set

Philosophy Final Exam Study Guide

View Set

Chapters 6-7 (Mastering Biology Questions)

View Set

BSG Quiz 1, BSG Quiz 1, Business Strategy Game Quiz 1, BUS 490 BSG Simulation Quiz 1, BSG Quiz 1

View Set

CMA 1-3 Statement of Cash Flows and Financial Statement Articulation

View Set

AP Psychology: Experiments & Research Methods

View Set

ACCTG 101 - Chapter 4 - Smart learning

View Set